Download as pdf or txt
Download as pdf or txt
You are on page 1of 77

Chapter 5

Linear Differential Equations


5.1 Introduction
Linear differenti~l equatio~s occ~r in the study of many practical problems in science and engineering.
Constant coefficient equations anse in the theory of electric circuits, vibrations etc. Variable coefficient
equations arise in many areas of physics, electric circuits, mathematical modelling of physical problems
etc. Some of the important variable coefficient differential equations are Bessel equation, Legendre
equation, Chebyshev equation etc. The solution of constant coefficient ·equations can be obtained in
terms of known standard functions. However, no such solution procedure exists for variable coefficient
equations. Often, we attempt their solution in the form of an infinite series. These solutions may
sometimes reduce to known standard functions.
A linear ordinary differential equation of order n, is written as
dny dn-ly dy
a 0 (x) -
dxn + a 1(x) 1 + ...
dxn- + an-i(x) dx + an(x)y = r(x)
or ao(X) y<n>(x) + a 1(x) y<n-l>(x) + ... + an-l (x) y'(x) + an(x) y(x) = r (x) (5.1)

where y is the dependent variable and xis the independent variable and ao(x) ~ 0. I_f r(x) = 0, then it
is called a homogeneous equation, otherwise it is called a non-homogeneous equatwn. For example,
a second order homogeneous equation is of the form
ao(x)y" + a 1(x)y' + ai(x)y = 0, ao(x) ~ 0 (5.2)
· Of th e form
· is
and a non-homogeneous second order equat10n
ao(x)y" + a 1(x)y' + a 2(x)y = r(x), ao(x) ~ 0. (5.3)

. linear second order constant coefficient


If a;(x), t. = 0, l, 2 are constant s then the equations are
equations. A few examples of linear second order equations are
2 X
y" + 4y' + 3y = X e , (5.4)
y" + 2y' + y = sin x, (5.5)

x2y" + xy' + (x2 - 4)y = 0, (5.6)


(1 _ x2)y" _ 2xy' + 20y = 0. (5.7)
5 ·2 Engineeri ng Mathemat ics
. ations and Eqs. (5.6), (5.7) are variable
Equatmn s (5.4), (5.5) are constant coefficient second order equ
coefficien t second order equations.

5.2 Solution s of Linear Differential Equations


. the interval may be open, closed,
We ~ssume that x in Eq. (5.1) varies on some interval/, ~here be valid for all x E (0, ex>) or
semi-ope n or infinit~. For example, the differential ~quatm~ ma~ all satisfy the equation. Hence,
t identlc Y .
x E (- 00 , 00). If y 1(x) 1s a solution of the Eq. (5.1), then it muS <n> ( ) must be contmuou s on 1.
d
Yt (x ) must be continuou sly differentiable n - 1 times an Yt x

We now state an important result regarding the uniqueness of solutions.


Theorem 5.1 If the functions ao(x), a,(x), ... 'an(x) and r(x) are continuou s over/ and ao(x) 0 *
on /, then there exists a unique solution to the initial value problem
(5.8)
aoy<n) + a,y<n-1) + ... + lln-Jy.' + llnY = r(x)
(5.9)
, ) - Y (n-l>(xo) = c n
y(x0 ) = c 1, y (x0 - C2, •.• ,

where Xo E /, and ci, c 2, . . . , en are n known constants.


This theorem does not give us a procedure to find the solutions but guarantee s that there exists a
unique solution if the conditions stated in the theorem are satisfied.
If the condition s of the Theorem 5.1 are satisfied, then the differential Equation (5.8) is said to be
normal on I (these condition s are both necessary and sufficient for the differenti al equation to be
normal).
Example 5.1 Find the intervals on which the following differential equations are normal.
(a) (1 - x2)y" - 2xy' + n(n + l)y = 0, nan integer.
(b) x 2y" + xy' + (n - x2)y = 0, n real.
2

4
(c) ./xy" + 6.xy' + 15y = In (x - 256).

Solution
(a) Here, aa(x) = (1 - x2), a 1(x) = - 2x, and a2(x) = n(n + 1). Now, a 0, a 1 and a 2 are continuou s
*
everywhe re in (- 00 , 00). Also, ao(x) = 1 - x2 0 for all x E (- 00 , oo), except at the points
x = _ 1, 1. Hence, the differential equation is normal on every subinterva l / of the open intervals
(- oo, - 1), (- 1, 1) and (1, 00).
2
(b) Here, a 0(x) = x2, a1(x) = x a2(x) = n -x2. We find that 0 o, a1 and a2 are continuou s everywhe re
2 *
in (- oo, oo). Also, ao(x) =x 0 for all x E (- , ) except atx = 0. Hence, the differenti al equation
00 00

is normal on every subinterval / of the open intervals (- oo, 0), (0, oo).
(c) Here, ao (x) = ../x, a!(x)_= 6x, a2(x) = 15, and _r(x) = !n (x4-256) . Now, ao, a1, a2 and r(x) are
continuou s for all x sat1sfymg x > 4. Hence, the differential equation is normal on every subinterval
J of the open interval (4, 00),

Remark 1
If the functions ao(x), a!(~), ... 'an(x) are continuous over/ and ao(x) ~ 0 on/, then the only solution
of the homogen eous initial value problem
(n) + (n-1) ,
aoY a,y + • • • + a,,_,y +any= 0 (5.10)
linear Differential Equations 5.3
Y(Xo) =0, Y(x.,) =0, ... , y<...l)(Xo) =0 (5.11)
~~~ E / ' is the trivial solution y =z
Yn o on I.
mbination of functions Let r ( )
Linear co JI x , f2(x), ... , fn(x) be n functions . Then
C1 /1(x) + c2 f2(x) + · · · + c, /.(x), where c,. c , ... , c, are constanls
. called a linear combination of the given functions . 2
1s
th
We had earlier defined at a function Y1(x) is a solution of a non-homogeneous or a homogeneous
ation. if the equation reduces to an identity when y (x) is substituted into it. Let y,(x), Y,(x), · · ·
1
equ(x) be m solutions of the Ii near homogeneous equation (5. I 0). Then, we show ;n the followin g th al
,y,,, superposition pri11ciple or linearity principle holds·
the

Theorem S.2 If Y1 (x), ~2(x~, · · · , Ym(x) are m solutions of the linear homogeneous equation (5. l 0)
on a/' then a linear· combmat1on
1 1 · of the solutions c 1y 1(x) + CiJi(x) + ... + c'"y'"(.x), where c1, c2, · · ·•
f Eq
c,,, rc constants 1s a so a so ut1on o . (5.10) on /.

proof Substituting the linear combination y = c 1y 1(x) + ciYi(x) + . .. + c,,,y,,,(x) into Eq. (5 . 10), we get
dn
ao (x) -dxn [c 1y 1 (x) + c 2y 2 (x) + . . . + c,,,y,,, (x)]

dn-1
+ a1 (x) ---:-
dxn-1 [c1Y1 (x) + C2Y2 (x) + • · · + c,,,y,,, ( x)]

+ ... + a 11 _1 (x) fx [c1Y1 (x) + C2Y2 (x) + · · · + c,,,y,,, ( x )J


+ an (x)[c1Y1 (x) + C2Y2 (x) + · · · + c,,,y,,, (x)]

dx" + a I (x) d"-1Y1


-- Ct [ ao ( X ) d"y1 d.xn-1 + . . . +an-I (x) ddyl
X + an (.x)y1]

d"y2
+ c 2 [ a 0 (x)-d ( ) dn-lY2
n + a1 x dxn-1 + . . . + an- 1(x) ddy2
x + a,.(x)y2]
X .

d n-1
+ ... + c,,, [ a 0 (x) d" y,,, + a 1(x) d.x"-1
dx" :Ym + ... + a,._ 1 (x) dy,,.
d.x + an ( x )Ym ]

= c1[0] + ci[0] + ... + c,,,[0] = 0 .


.
since Yi(x), Y2(x), ... ' y,,,(x) are solutions of the linear homogeneous equation.

Remark 2 . hold for a non- homogeneous equation or a nonlinear equation.


Superposition principle does not . b' lion c , -, + c,e' are solutions of the
Example S.2 Show that e ' e -x x and their hnear com ma t
homogeneous equatmn. Y,, - y-0 - ·

_ ~ 0.
Solution For y = e-x, we have Yt, -- - e-x , y;' = e - x ,,• y;' - Yt
I , x ,, = es • Y2 - Y2 - 0.
For Y2 = e x 'we have Y2 = e 'Yi
Hence, e-x and ex are solutions O f· YII - y : 0.
5.4 Engin eerin g Math emati cs

Subs tituti ng y = c 1e-x + c 2ex = Ci Yi + c2 y 2, we obtain


,, Y2) = (0) + (0) = 0.
y " -y = <c1Y1 + C2 Y2)" - (C1Y1 + C2Y2) = C1 ( YI" - y l ) + C2 ( Y2 - C1 C2

5.2.1 Linear Independence and Dependence


.
Let /1 (x), /2(x) , ... , fn(x) be n functions. Then, these funct . said to be linea rly indep ende nt
ions are
on some interv al / (whe re they arc defined), if the equat ion

c 1 / 1(x) + c 2 fi(x) + , • • + Cn fn(x) = O (5.12 )


implies Ct = Q = c 2 = .. , = Cn.
Thes e funct ions are said to be linea rly depen dent on /, if Eq.
all zero. In this case, one or more functions can be expre ssed
(5.12) hold_s fo_r ci' ; 2 h· · · '
c_n ~ot
as a linea r comb mat1 0n ° t e rema mmg
funct ions. For exam ple, if c *
1 0, then
/1 (x) = - - 1 [c2'2 (x) + ... + Cnfn (x)].
C1
Conv ersel y, if any funct ion/; (x) can be expressed as a linea .
r comb inatio n of the func tions /1,h, · · ·
f i-1 • h+1 • • • , In then the given set of functions are linear
ly dependent.
Exam ple S.3 Show that the funct ions/ i(x) = x2,'2 (x) = x3.'3
(x) = 6x 2 -x 3 are linea rly depe nden t
on any inter val /.
Solu tion We have /3(x) = 6x 2 -x 3 =6/ (x)-/ i(x). Hence, the
1 given funct ions are linea rly depe nden t
on any inter val /.
Exam ple S.4 Show that the functions x 2 - 1, 3x2 and 2 - 5x 2
are linea rly depe nden t.
Solu tion The given funct ions are linearly dependent if the equat
ion
c 1(x 2 - 1) + c 2 (3x 2) + c 3(2 - 5x 2) = 0
(5.13 )
holds for c 1, c 2, c 3 not all zero. We have from Eq. (5.13)

(c1 + 3c2 - 5c3)x2 + (2c3 - c 1) = 0, for all x.


We have , c 1 + 3c2 - 5c3 = 0, and 2c3 - c1 = 0. The solution
of these equa tions is c 1 = 2c , c = c
wher e c 3 is arbitr ary. For exam ple, if c3 = 1, then c1 = 2, c2 3 2 3
= l andfJ (x) = - 2/1(x) - f (x). The given
funct ions are linea rly depen dent. 2

Exam ple S.S Show that the functions x, x 2, x 3 are linearly indep
enden t on any inter val /.
Solu tion We have / 1(x) = x, fi(x) = x 2, /3(x) = x 3. . Subs
tituti ng in equa tion (5.12 ), we get
cix + c 2x2 + c 3x 3 = O. For findin g the values of the three
const ants, take three distin ct arbit rary point s
xo, x 1, x 2 (;t: 0) on/. Henc e
C1Xo + c2xi + c3xJ = 0, C1X1 + C2xr + C3xl = 0, C1X2 + C2Xi
+ C3Xi = 0.
This syste m of homo gene ous algeb raic equat ions has a non-t
rivial solut ion if the deter mina nt of
the coeff icien t matri x vanis hes, that is

Xo x20 x30
det = X1 x2
1 XJ
1 = o.
X2 x22 X23
Line ar Diff eren tial Equ atio ns 5.5
£valuating the dete rmi nan t, we hav e

det = (x1 - Xo)(x2 - xo)( x2 - x1)xox1x2,


*
Since Xo, xi,_ x2 are di~tinct, det 0. The refo
the given func tion s are line arly inde pen den re, the only solu tion is c = 0, c = 0, c3
1 2 = 0. Hen ce,
t.
The pro c~u re use d in Exa mpl e 5.5 is leng
thy and a diff icul t one . It is not alw ays pos
examine the hne ar dep end enc e or inde pen sibl e to
den ce in this way. A very eleg ant proc edu
linear inde pendenc e or dep end enc e of a give n re to test the
set of functions is the app lica tion of Wronski
J1(x), fi(x ), · · · • fn(x ) be n fun ctio ns. The Wro ans. Let
W(f1, I 2, .. • , fn) and is defi ned by nsk ian of thes e fun ctio ns is den oted by

/1 /2 In
f{ /2
W( /1,/ 2 • · · · ,fn) = 1; (5.1 4)
= W(x )•
it-•> /in-1) Jjn-1)
The Wro nsk ian of the n func tion s exis ts if
all the func tion s / , Ji, ... , In are diff eren
times on the inte rval /. If any one or mor e 1 tiab le n - l
func tion s are not diff eren tiab le then the Wro
not exist. nsk ian doe s
We hav e the foll owi ng resu lt for test ing the
line ar dep end enc e or inde pen den ce of the
the linear hom oge neo us diff eren tial equ atio solu tion s of
n (5.10).
Theorem 5.3 If the coe ffic ient s a (x)~ a
0 1(x), ... , an(x) in the line ar hom oge neo us equ atio
n
aoy(n) + a1y(n-1) + • • • + an- lY' + any = O,
are con tinu ous on I and y 1(x), y (x), ... ,
*
ao O (5.1 5)
2 Yn(x) are n solu tion s of this equ atio n, then

on I,
*
(i) W(x) = W(y i, y 2, ... , Yn) 0 for all x
e / ~ y 1(x), Y2(x), . .. , yn(x) are line arly
inde pen den t
(ii) W(x0 ) = 0 whe re Xo e / is any fixe d poin
t, imp lies W(x ) = 0 for all x in I and the fun
y 2(x), ... , yn(x ) are line arly dep end ctio ns y 1(x),
ent.
Proof Let y 1(x), y 2 (x), . .. , yn(x) be line arly dep
end ent on/ . By defi niti on, ther e exis t con
c2, ...• en not all zero , suc h that stan ts c 1,

c 1y 1(x) + c 2 y 2(x) + ... + Cn yn(x) = 0, for


all x E /. (5.1 6)
Differentiating Eq. (5.1 6) succ essi vely , n
- 1 time s, we get
c 1y 1(x) + C2Y2 (x) + , . · + CnY ~ (x) = 0

c 1 y 1(x) + C2Y2(x) + . , , + Cny ;(x) = 0

c1 y?- l)
(x) + C2Y2(n-1 ) (x ) + · • · + CnYn(n-1 ) ( X ) = O
. (5.1 7)
Eqs . (5 .16) , (5. l ?) form a hom oge neo us, line
ar syst em of alge ~rai c equ ~ti~ ns. Non-triv
of the syst em exis t if and onl y if the dete ial solu tion s
rmi nan t o~ the coe ffic t~nt mat nx •~ zero
this dete rmi nan t is the Wro nsk ian W(x ) of for all x e /. But
the solu tmn s. Hen ce, 1f the solut10ns are
W(x) = 0 for all x E /. dep end ent then
5.6 Engin eering Mathe matic s
h stem of equat ions given by (5.16) ,
Let now W(x0 ) = 0 for some fixed point xo E /. Then, t ~ sy
(s · 17) h as a nont.n.vinl II zero. Hence , y*(x) = c~ y (x)
. .
• solutm n c 1 = c,• . c2 = c2,
• c
···• " = c
"
not a 1
Equa tion (5.15 ). Using
r
+ C2Y2 (x) + ... + c ~y ,, ( x) is a soluti on of t~e mear . homo geneo us
itial condi tions y*(xo) = 0, (y*)'( xo) = 0,
Eqs. (5 . 16) and (5 . 17), we find that y•(x) also satisfi es th e 10
d ' tions form a homo geneo us
· · · • (y•)<"- 1>(.,·o) = 0 . Now, the differ ential equati on (S. lS) a nd _tl~else clon 1
· · • p roblem Since the soluti on
1mtrnl value probl em . Hence , y*(x) s O is the soluti. on o f t he 10 · 1tia va ue ·
( ) _ o or for all x
o f the initia l value probl em is uniqu e, we o b tam .
Y *(x) -
- y X - ' ' '
• • . d
Ci Y 1 (x) + c 2 ,v 2 (x) + . .. + c ~y ,, (x) = O, where not all c, are zero. Hence the soluti ons are epend ent.
s·mcc .'<) 1s· arb'1trary. W(x.o) = O for some x~ E / · 1· W(x) - 0 for all 'x E /.
imp 1es -
We now defin e the gener al soluti on of the homo geneo us Eq. (5. 15
)-
Th
eorem 5.4 lf the coeffi cients a (x). a (x), . . . , a,,(x), ao(x ) ~ o• 1·n the linear homo geneo us equat io n
0 1
(5.1 5) are contin uous on / , then the equati on (5 . 15) has 11 linear
."2(x). · · • • y,,(x) are 11 linear ly indep enden t soluti ons,
f
ly indep endent so_1u t~ns. . Yi
then th~ g~ner al soluti on is given Y
If (

y(x) = c, Y1(x) + c y (x) + . .. + c,, y ,,(x), that is, their linear
2 2 comb matio n .
The n linear ly indep enden t soluti ons Y1 (x), Y2(x), .... ' ynCx)
are also. called the Junda m~nta l
soluti ons of Eq. (5. 15) on I. This set of funda menta l soluti ons forms
a basis of the n th 0rd er linear
homo geneo us equat ion.
Exam ple 5.6 Show that the functi ons x, x 2 , x 3 are linear ly indep enden t on
any interv al / , not
conta ining zero (see Ex.am ple 5.5).
Solut ion The Wron skian of the functi ons is

X xi xJ

W(x) = 1 2x 3x 2 = x(l2x 2 - 6x 2 ) - (6x 3 - 2x 3 ) = 2x 3 •


0 2 6x
There fore, W(x) ~ 0 on any interv al not contai ning zero. Hence ,
the functi ons are linear ly indep enden t
in (- oo, 0 ), (0, oo).
Exam ple 5.7 Show that the functi ons 1, sin x, cos x are linear ly indep enden
t.
Solut ion The Wron skian of the functi ons is

1 sinx cosx
W(x) = 0 cosx -sinx = -1.
0 -sinx -cos .x
Henc e, the given funct.ions are linear ly indep enden t on any interv
al /.
Exam ple 5.8 Show that e·C, eix, e1r are the funda menta l soluti ons of y'" _ 6y"
any interv al /.
+ l ly' _ 6y = o. on
Solut ion Subst itutin g y = e ". e2.r, e1r, we find that they satisfy the differ
ential equat ion
y'" - 6y" + l ly' - 6y = 0.
The Wron skian of these functi ons is
Linear Differential Equ atio ns 5.7

ex e2x e3x
1 1 l
W( x) = ex 2e2 x 3e3x = e6 x 1 2 3 = 2e 6 x -1:- 0·
ex 4e2x 9e3x 1 4
9
Therefore, the solu tion s are line arly inde pen
den t and they form a set of fund ame ntal solu
interval /. tion s on any

Example 5.9 Sho w that the set of func tion s (x,


1/x} form s a basis of the equ atio n x 2 y " + xy'
Obtain a part icul ar solu tion whe n y(l) - Y = 0.
= 1, y'(l ) = 2.
Solution We hav e

Yi (x) = x, Y1 = l, y;' = 0, and x 2 y;' + xy 1 - y 1 = x - x = 0

Y2( x)= l/x, y 2 =- llx 2 ,y =2 lx 3


2
Y2 = x 2( }3) + X ( - /2) - ( ~) = 0.
2
and X Yi + XY:z -

Hence Y1(x) and Y2(x) are solu tion s of the


give n equ atio n. The Wro nsk ian is give n by

1/x 2
= -- -1:- 0, for X ~ 1.
-1/x 2 X

Therefore, the set {y 1(.x); y (x)} form s a basi


2 s of the equ atio n. The gen eral solu tion is

C2
y(x ) = c 1y 1 (x) + C2Y2 (x) = c1x + - . X
Substituting in the give n con diti ons , we get

y(l) = 1 = C1 + C2, y'(l ) = 2 = C1 - C2.


Solving, we obta in c 1 = 3/2, c 2 = - 1/2.
The part icul ar solu tion is y(x ) = 1(3x - !)•
Exercise 5.1
From the following line ar differential equation
s, find the constant coefficient and variable coef ficie nt equa tion
s.
1. y" _ a2y = o. 2. y' = ylx.
3. y"' + 3y" + 6y' + 12y = x2. 4. x 3y'11 + 9x 2y 11 + l8xy ' + 6y = 0.
5. (I - x)y" + xy' - y = 0. 6. y 11 -(l +x 2)y= 0.
Find the intervals on whic h the following
differential equations are normal.
1. y' = 3ylx. 8. (1 + x 2)y + 2ty' + .Y = 0.
11

9. x2y" - 4xy ' + 6y = x. 10. y"+3y'+ ✓xy = sinx .


11. y"' + 9y' + y = log (x2 - 9). 12. .Y,, + I ,r I y' + Y = x In .r .
5.8 Enginee ring Mathematics _ In sin (nx/4).
H '+ 6y-
,, , _ 14. Y + xy ) and satisfies the conditio ns y(O) == o,
13. x(I - x)y - 3xy - Y - 0 · e (0 00 l · ?
. x2 n + , _ 4y = 0 , x ' . eness of such a so ut1on . Is the
15. Verify that y = x2 is a solut1on of Y xythe existence and uruqu
y'(O) = O. Does Theorem 5.1 guarantee · ..
Remark 1 applicable in this case? o0) which satisfies the c~nditlo ns y(O) == 0,
00
• -2 ,, , _ y = 0, x E (- • f such a solution?
16. By inspection find a solution of ry + xy e and uniquene ss 0
th existenc
y'(O) = 2. Does Theorem 5.1 guarantee e
17. Show that 2 X3 -3 5 X 5 0,
X - '

Y1(x) = x3 - x2, - 3 :5: x :5: 3, and Y2 (x) = { x3 - x2' 05 x ~3


. . n , = 0 and the condition s y(2) =4, y'(2) =8. But Y1(x),
6
both satisfy the differential equation x2y - 4xy + Y
yi(x) are different. Does this contradict Theorem 5. 1? . .
. d·n ential equation . Venfy also that a lmear
Verify that the given functions are solutions of the associated i er
combinat ion of these functions is also a solution.
18. 1, X, ex; y"' - yn = 0.
20. e-x cos 2x, e-x sin 2x; y" + 2y' + 5y = 0.
Examine whether the following functions are linearly independent for x E (0, oo).
21. 2x, 6x + 3, 3x + 2. 22. x2 - X, 3r + X + 1,
2
X + · 9r -
23. x2 - 2x, 3x2 + x + 2, 4x2 - x + l. 24. sin x, sin 2x, sin 3x.
25. 1, cos x, sin x. 26. ex, sinh x, cosh x.
27. x2, llx2. 28. In· x, In x2, In x3.
29. x - l, x + 1, (x - 1)2• 30. e-x, sinh x, cosh x.

31. Find the intervals on which the three functions 1, cos x, sec x, x > 0 are linearly independ ent.
32. Determin e how many of the given functions are linearly independ ent on [O, l].
(i) 1, 1 +x, x2,x(l -x),x; (ii) 1 +x, 1-x, 1,x2, 1 +x2.

33. Show that y 1(x) =sin x, and Y2(x) = 4 sin x - 2 cos x are linearly independ ent solutions of y" + y =O. Write
the solution y3(x) = cos x as a linear combination of y 1 and y 2• .

34. Let ~(x)y" + a 1(x)y' + ai(x)y = 0 be a second order differential equation . Let ao(x), a 1(x), ai(x)
be
the
continuo us and ao(x) * 0 on I and Y1(x), Y2(x) be two linearly independ ent solutions . Show that
th t
Wronskia n of y ·1(x),• y 2(x)b satisfies the differential equation ao(x)W'( x) + a i (x)™(x ) = o·. Al so, sh ow a
the Wronski an 1s given y

W(x) = c e -f (a1(x)lao(x)Jdr_
(This is called the Abel's formula).
35. Show that cos al, sin al are solutions of the equation yn + a2y = 0 a* 0 .
they
are independ ent. Use the result (Abel's formula) given in Pr0 bl ' 34 on any mterval. Show that
em and find the Wronski an. Are the
two Wronskians same?
36. Show that e2x and xe2x are solutions of the equation y" _ 4y' + 4 _ .
are
independ ent. Use the result given in problem 34 and find th WY - O ~n any interval. Show that they
. . b e ronskian. Arc the two Wronski ans same?
. ·
Show that m the following pro lems, {y1(x)} forms a set of fund
amental solut1ons (basis) to the correspo nding
different ial equation .
2
37. x I14 , x 514 ; 16x y" - Bxy' + 5y = 0, x > O.
Li11 t!ar D(ff~ re11tial Eq uatiofl s 5.')

38. e2.r cos 3x, eir sin 3x; 2yH- Sy' + 26y = 0.
9, 2
3 l. x ; :?y H- xy' = 0, x > 0.
40, ex• el.r• e- 3x.• )' ' H - 7y' + 6y -- 0·
41, ex. ex cos x. ex sin x: y"' - 3yH
+ 4y' - 2y c 0.
42, e2X. e-.t cos (.fix) . e-x sin (-./L:
): .vm - 8y ::: 0.
43. sin (ln x2) . cos (ln x 2): xlyH + xy'
+ 4y = 0, x > 0.
44. Let the coe fficients a0(x) , a (x), a (x)
1 2 In the equ atio n " o(x)y" + "i(.x)y' + a (x)y
a0(x) 'I- 0 on /. Let ly,(x) . Y2(x)
} be the basis (se t of fund nment 2 = 0 be continuous and
the set lu(x). v(x ) } such that" = a nl solu1 ions) of 1hc equalion . Show 1hat
y1(x) + by2(x), v = cy (x) + dy (x).
if ad - be 'I- 0. lf y,(x) = cosh h , y 1 2 Is also n bas is of the equation
2 = sinh h , obtuin a simple form of II nnd
~ u.
"5. Let y,(x), yi(x) be the linearly ind
ependent solutions of the equ ation y"
that there is no point xo E I at which + a(.x)y' + b(x)y = 0 on /. Show
(i) both y1(x) , y2(x) vanish , (ii) both y
46, Let {y, (x). Y2(x)} be the bas 1(x) . yz(x) taJcc eJtl remc values.
is of the equation y" + a(x)y' + b(x
written as the Wronskiari W(y , Yi, y )y = 0. Show that the equatio n can
2 ) = 0. be
47. Let y1(x) be a solution of the hom
ogeneous equation y" + a(x)y' + b(x)y
The coefficients a(x) arid b(x) are con = 0, on the interva l/ : a Sx Sf'>.
tinuous on /. If the curve y = y (x) is tan
x1 in I, then prove that y (x) =0. 1 gential to the .x-axis at a point
1
Using the problem 46, find a differentia
l equation of the form y" + a(x)y' +
functions are solutions. b(x)y = 0 for whi ch the follow ing

5.3 Methods for Solution of Line


ar Equations
In this section, we shall discuss
various methods of finding soluti
define the differential operator D. on of linear equations. We first

S.3.1 Differential Operator D


Sometimes, it is convenient to wr
ite the given linear differential equ
differential operator D = dldx. We ation in a simple form using the
define an operator T as a transform
a function/ in V into another functio ation T: V ➔ W that transform s
n T(V) in W. Let the operator D be
differentiable functions f on /, by defined, over the set V of all
D = dldx. 1

Then, we write
Df (x) =DJ= (Df)(x) = dx =f'. (5 .18 )
We have, for example D( x") = ! (x 11 ) =nx 1 1
n constant~ D(cos x) = - sin x, etc
•- ,
.
Let f(x) and g(x) be differentiable
functions . Since D is a linear ope
rator, we have
D( af + bg) = aD f + bDg, a, b
constants.
We also have for f e V , the set of fun
2 ctions having a sec ond deriva tive
on I

D(Df) = D( f') = :\. (J' ) =/ ".


We simply write D(Df) = D(D)/ = D2/ so that
5.10 Engineering Mathematics
k - f (k)
D 3(f) = D(D2f) = D(f ) =J"'' . .. ' D (j) -
II .

where/is sufficiently differentiable. (/)-/,we have D0 (f) = l (f ) ==f


·
We define D = 1, so that if 1 is the operator
0 defined by 1 - '
We, now define the operator L by
dn dn-1 (x) .JL + an(x)
L = ao(x) - + a1(x) dxn-1 + . . . + an-I dx
dxn
(5 .1 9)
= a (x)Dn + a (x )D n-1 + . • • + an-1 (x)D + an(X) = P(D)
0 1

which is a polynomial in D, so that

dn dn-ly (x) dy + an(x)y


Ly= a 0 (x) dx
. dxnY + a 1( x )
__ - ~ +···+an-I
dxn-1

= ao(x)Dny + a1(x)Dn- 1y + ... + an-1(x)Dy + an(x)y


= [a0(x)Dn + a 1(x)Dn-l + ... + an-1(x)D + an(x)]y = P(D)y. (5.20)

For example, the differential equation

d2y + 5 dy + 6y = 0
dx 2 dx
can be written as
Ly= (D2 +SD+ 6)y = 0 (5.21)
where the operator L is given by L = P(D) = D2 + SD + .6.
Similarly: the equation
d2 d
--1...
2
+ 2 -2'.. + 2y = x 2
dx dx
can be written as

Ly = (D 2 + 2D + 2)y = x2 (5.22)
where the operator L is defined by L = P(D) = D2 + 2D + 2.
Suppose y = r. Then, D(y) =D(~ =memx and D2(y) =m2fl"X. Substituting in Eq. (5.21), we obtain
2
Ly = (D + SD + 6)y = (m2 + Sm + 6)y = P(m)y
using Eq. (5.19). Therefore,
2
P(D)y = (D + SD + 6)e"'X = P(m)y.
In general, substituting y = fl"X in the equation (S.20), we get
P(D)y = (aoDn + a1D n-1 + ... + an-1D + an)e mx
n n-1
= ( aom + a1m + ... + an- 1m + an)emx = P(m)emx.
(5.23)
When a,, i = o, 1, ... , n are constants, the operator L = P(D) can often be factorised.
For example, we have
(i) D 2 + SD + 6 = (D + 2)(D + 3)
2
(ii) D3 - 6D + llD - 6 = (D - l)(D - 2)(D - 3).
Lin ear Diff eren tial Equ atio ns 5.11

D(x 2y), or in
ofte n not possible. For example, x2D y '#
When a, are fun ctio ns of x, f~ctorisati~n is
hand side is D[a(x)y] = a(x )y' + a'(x )y.
general a(x) Dy' # D[a (x)y l, smc e the ngh t
ients
ogeneous Equations with Constant Coeffic
s.J.2 Solution of Second Order Linear Hom
er equ atio n
Consider the line ar hom oge neo us seco nd ord
(5.2 4)
ay" + by' + cy = 0, a, b, c are constants.
n as
In the operator nota tion , we wri te the equ atio
(5.2 5)
= 2
Ly= P(D )y aD y + bDy + cy =(aD 2 + bD + c)y = 0.
cha pter , we hav e sho wn that the solu tion of the first ord er equ atio n y' +m y= 0 is
In the prev ious
solu tion of the equ atio n y' - my = O is y = emx + c. The refo re, it is natu ral to try
y:: e-mx + c; and the stan t to be
, for Eq. (5.25), whe re m is an unk now n con
for a particular solu tion of the form y2 = emx
we obta in from Eq. (5.24)
determined. Sin ce y' = memx, y" = m emx,
(am 2 + bm + c)emx = 0.

Since emx '# 0, we obt ain


(5.2 6)
am2 + bm + c = 0.
equ atio n
is an alge brai c equ atio n in m. It is call ed the cha rac teri stic equ atio n or the aux ilia ry
This g y"
ar hom oge neo us equ atio n (5.2 4) (we can wri te the characteristic equ atio n by repl acin
of the line bein g
2
y' by m and y by 1 in Eq. (5.2 4) imp licit ly noting that solutions of the form ra re
by m , equ atio n
ed). The roo ts of this equ atio n are call ed the cha rac teri stic roots. The qua drat ic
determin
(5.26) has the roo ts
m = [-b ± .Jb 2 - 4ac ]/2a .

We have the foll owi ng three cases.


(i) The roo ts are real and dist inct , say m = m 1, m 2 ; m1 * 2
m 2 if b - 4ac > 0.
2
mi if b - 4ac = 0.
(ii) The roo ts are real and equ al, say m = mi,
2 0.
(iii) The roo ts are com plex if b - 4ac <
three cases, we proceed as follows.
To find the com plet e solu tion in the abo ve

Real and distinct roots


= • The n, we obta in two solu tion
s of the equ atio n (5.2 4) as
Let the dist inct roo ts be m = m 1 and m m 2
sinc e the Wro nsk ian,
emix and em x. The two solu tion
2 s are linearly independent on any interval /,

+m 2)x~ o.
= (m 2 - m. 1 ) e (m1 ~

Hence, the general solu tion of Eq. (5.24) is


(5.2 7)
5.l ~ Er.gir.eaing .\Jc;1han:.uic·s
. ,, v' - 6v = O.
-~ . tial equauon Y - - ·
ExampJe S..10 Find the solution of the d,w•.ren . .
. ·h cteristic equauon ns
Solution Substituting -"= e'u. we obt-am th~ c ar.i r m = - 2. 3.
m 1 - m - 6 = 0, or (
m - 3)(m + 2) = 0. o . .
. .\r • d e-!r. The general soluuon ,s
The two linc3.fly independent solutions rue e 30
_\{.t ) = Aelr + Be-:.".

E.nmple S.ll SolYe the initial value problem


4y"' - S_v' + 3y ::: 0. ){0) = 1. y'(O) = J .
. the ch·arac teristic equation ns
Solution Substituting y = e'".r. we obtain ·
4m2 - Sm + 3 = O. or m. = 1/2. 3/2.
·
Hence. the linearly independent soluaons r12 d (1r)l2 The general solution is
are e· an e ·
y(x) = Ae (lr)/2 + Be.rr...
Substituting the initial conditions, we get

y(0) = 1 =A+ B. y '(O) =? =23A + B


2.
Solving the above equations, we get A = 512 and B = - 3/2. The solution of the initial value problem is
y(:c) = [5e(3.xY2 - 3e.r'712.

Real and equal roots


Real and equal roots are obtained for the characteristic equation (5.26) when b2 -4ac = 0. In this case
the repeated root ism=:= - b/(2a). This value gives one solution as y (x) = emx = e-(bx>1<2a>. We need
1
to determine another linearly independent solution yi(x). so that {y 1(x). y (x)} forms a basis for the
2
equation. The second solution Y2(x) can be determined in a number of ways. We shall show that if 111
is a repeated root then em." and xem:c are the two linearly independent solutions.
For y 2 (.x) = xemx. m = - b/(2a). we have

Y2 =xem.r • Y2 =(1 + mx)em.r. y!f. =(2 + mx)mem·" .


Substituting in Eq. (5.24). we get
[ma(2 + mx) + b(l + mx) +. cx]emx = o
or [(2ma + b) + (am 2 + bm + c)x]em.t = O.
2
Since _2ma + b =- b: b =?
a nd am": + bm +m: = O, ~is equation is automatical ly satisfied. Therefore,
xemx 1s also a solution. Smee e and xe are linearly indepe d h
fundamenta l solutions. Hence, the general solution is n ent, t ey form a set of the two

y(x) = Aem.r + Bxemx = (A + Bx)emx. m = - b/(2a). (5.28)


Alternative We can use the following method (reduct • if
independen t 'solution. wn ° oroer) to find the second linearly
Let
Linear Differential Equations 5.13

where y 1(x) = em", m = - b/(2a) be a solution of Eq. (5.24). We have

Y2I _- uy1I + u I Yi, Y2II = uy,II + 2 u /Yi+


/ II
u Yi•
Substituting in the differential equation, we obtain

a(uy,' + 2u'y 1 + u"y 1 ) + b(uy 1 + u'y 1 ) + cuy 1


= ay1u"+ (2ay 1+ by 1 )u' + (ay 1'+ by 1+ cy 1 )u = Q. (5.29)

Since y 1(x) is a solution, we have ayt' + by 1 + cy 1 = 0.

Also, 2ayi + by1 = 2a( - 2ba )e-f/Jx)/(2a) + be-<bx)tan> = 0.


Hence, Eq. (5.29) reduces to ay 11/' = 0. Since a -:f. 0, y 1 -:f. 0, we get u" = 0, whose solution is
u = c1x + c2, Therefore, Y2(x) = (c 1x + c 2)y 1(x) = c 1xy 1(x) + c y (x). Since y (x) is a solution, the
2 1 1
second linearly independent solution is xy 1(x), (note that a linear combination of the two linearly
independent solutions is also a solution). The general solution is
y(x) =Ay 1(x) + Bxy 1(x) = (A + Bx)emx, m =- b/(2a)
which is same as Eq. (5.28).
Alternative The second linearly independent solution can be determined by factorising the differential
operator and reducing the given second order equation to a first order equation. We have

(aD 2 + bD + c)y = a[ D 2 + : D+ ~ ]y = 0, a -:t- 0.

Since b2 - 4ac = 0 and m = m 1 = - b/(2a) is a repeated root, the operator is factorisable so that we
can write the equation as

(5.30)
Set (D - m 1)y = u. Then, Eq. (5 .30) reduces to (D - m 1)u = 0 or u' - m 1u = 0 whose solution is
u = c I emix. Substituting in the equation (D - m1)y = u, we obtain

(D - m1 )y = y' - m1Y = u = c 1em 1x.

The integrating factor of this equation is e-m ix . Therefore, the solution of this equation is

ye
-mix
= J C1e
mix -m1xdx
e + C2 = C1X + C2

or mix
y = ( C1X + C2 ) C
Which is same as y 2(x) obtained in the previous case.
Example 5.12 Find the solution of the differential equation 4y" + 4y' + y = 0.
Solution Assume a solution of the form y = emx. The characteristi c equation is given by
4m2 + 4m + I = 0, or (2m + 1)2 = 0, or m = - 1/2, - 1/2,
Which is a repeated root. Hence, the general solution is y(x) = (A + Bx)e-:cn.
5.14 Engineering Mathemat ics

Example 5.13 Solve the initinl value problem


y" + 6y' + 9y = 0, y(0) = 2, y'(O) C 3.
, . "'r Tl ,· rucu,rlstic equation is gi vcn by
Solution Assume u solution of the torm y = e · . 1e c rn
2 3
m 2 + 6m + 9 = 0, or (m + 3) = 0, or m = - 3, - ,
.ch · t • · ( ) (A + Bx)e- lt. Substituti ng in the initial
wht 1s a repeated root. he general so1uuon 1s Y x =
condition s. we get
= 2 =A. y' = Be-J.r - 3(A + Bx)e-1t, y'(O) = 3 = B - 3A.
y(O)
The solution is A = 2. B = 9. The solution of the given initial value problem is y(x) = (2 + 9x)e-J.c_
~ample 5.14 Factorisin g the differenti~I operator and reducing it into first order equations , solve
the differenti al equation y" - 4y' - 5y = 0.
Solution In the operator notation, the differentia l equation can be written as
(5.3 1)
(D2 - 4D - 5)y = 0, or (D - S)(D + l)y = 0.
Set (D + l)y = u. Then, we obtain from Eq. (5.31), (D - S)u = 0 . This is a first order equation whose
solution is u = Aes.i-. Hence,
(D + l)y = Ae 5x.

This is a first order linear equation, whose integrating factor is ex. Hence, we have the solution as

ex y = f Ae6x dx + B = t e6x + B,
or y = .!1_ e.sx + Be-x = Ce 5 x + Be-x
6
where C = A/6 is an arbitrary constant.
We could have written Eq. ·(5.31)" as (D + l)(D - S)y = 0 and obtain the same answer.
Example 5.15 Factorisin g the differentia l operator and reducing it to first order equations , solve the
differenti al equation 4y" + I2y' + 9y = 0
Solution In the operator notation, the differential equation can be written as
2
(4D + 12D + 9)y = (2D + 3)2y = 0. (5.32)

Set (2D + 3)y = u. Then, we obtain from Eq. (5.32),


(2D + 3)u = 0.
The solution of this equation is u = Ae-<Jx>n. Therefore I

(2D + 3)y =A,-<"> 12


, or (D+ ;)y = ~ ,-<Jx>12.
This is a linear first order equation whose integrating factor is e<1(}12. Th e so 1utton .
. .ts given by

ye
C3x >l 2 -
-
f2 A dx
+
B Ax
=T + B, or y = (Cx + B)e-<J.r>n,

where C = A/2.
linear Differential Equation.v 5.15

ColllPlex roots
When b2 - 4ac < 0, then the roots of the characteristic equation (5.26) arc complex. We have

-b ± ✓b 2 - 4ac -b ± ;.J4ac - b2
m= 2a = 2a = p ± iq

where p = - b/(2a) and q = .J 4ac - b 2 /(2a). Since the characteristic equation (5.26) has real
coefficients, the complex roots occur in conjugate pairs and are of the form p ± iq. Then, the solution
of the equation can be written as
y(x) = Ae<P + lq)x + Be(p-lq)x = Ae pxelqx + BeP'e- lqx = (Ae 111" + Be 111:.i)e p.c
= [A(cos qx + i sin qx) + B(cos qx - i sin qx)lepx
by the Euler formula. Simplifying, we obtain

y(x) = [c 1 cos qx + c 2 sin qx]ePx (5.33)


where c 1 = A + B and c 2 = i(A - B). Therefore, the two linearly independent solutions are
y1 = ePx cos qx and y 2 = ePx sin qx. The Wronskian is given by

_ e px cos qx eP" sin qx


W( Y1, Yz ) - = qe2px 1; 0
ePx (p cos qx - q sin qx) ePx (p sin qx + q cos qx)
showing that y 1(x) and y 2(x) are linearly independent.
Example 5.16 Find the solution of the differential equation y" + 2y' + 2y = 0.
Solution Assume a solution of the form y = emx. The characteristic equation is given by

. -2±,F 4 . .
m2 + 2m + 2 = 0, or m = = - l ± i = p ± iq.
2
The general solution is
y(x) = (A cos qx + B sin qx)e px ={A cos x + B sin x)e-x.
Example 5.17 Find the solution of the initial value problem
y" + 4y' + 13y = 0, y(0) = 0, y'(0) = l.
Solution Assume a solution of the form y = emx. The characteristic equation is given by

-4 ± ✓ 16 - 52
m 2 + 4m + 13 = 0, or m= = - 2±3i = p±iq.
2
The general solution is given by
y(x) = [A cos qx + B sin qx]ePx = [A cos 3x + B sin 3x]e-~.
Substituting in the initial conditions, we obtain
y(O) =0 =A,
y'(x) = Be-2x(3 cos 2x - 2 sin 3x), y'(O) = 1 = 38, or B = 113.
The solution of the initial value problem is
5.16 Engineering Mathematics

y(x) = ([21 sin 3x)/3.


. . f the boundary value problem
Example 5.18 Find all the non-trivial solutions, if any, 0
y" + w1 y = o, y(0) = 0, y(l) = O. . . .
mx Th haracteristic equation 1s given by
Solution Assume a solution of the form Y = e · e c
2 + w2 - o or m :::: ± i w.
m - '
The general solution is
y(x) = A cos wx + B sin wx. (5.34)

Substituting in the boundary conditions, we obtain


y(O) = o = A, y(l) = 0 = B sin (wl).
If B = 0, then we obtain the trivial solution y = 0.
For B ~ 0, we get sin wl = O = sin nn, n = 1, 2, ...
Therefore, w = nn/l. The general solution is
yn(x) = Bn sin [(nnx)ll], n = 1, 2, . • •
where B; 's are arbitrary. There are infinite number of solutions. Since the boundary value problem is
homogenous, by the superposition principle, the sum of these solutions is also a solution. Therefore,
the general solution is given by
00

y(x) = L Bn sin (nnx)


n=l
-
l
·

(The convergence of such an infinite series called the Fourier series, is discussed in chapter 9.)

5.3.3 Method of Reduction of Order for Variable Coefficient


Linear Homogeneous Second Order Equations
Suppose that we know one of the solutions of the second· order equation
ao(x)y" + a1(x)y' + a2(x)y = 0, a 0(x) :t:. 0 on/. (5.35)
Then, we can obtain the second linearly independent solution by the method of reduction of order.
Let y = y 1(x) be a non-trivial solution of Eq. (5.35), that is

(5.36)
Then, we write ~e sec~nd solution as y2(~) = u(x)y 1(x). Since u(x) = yi(x)/y 1(x) is not a constant,
y 1 and y 2 are two linearly independent solutions of Eq. (5.35). Now,
,_ I + I d II II
Yi - u Y1 uy1, an Y2 = u Yi+ 2u'y 1+ uy['.
Substituting in Eq. (5.35) and collecting the terms, we get

a 0 (x)y 1u" + [2ao(x) Y1 + a1(x)yi]u' + [a 0(x) y;' + a (x) y + a (x)yi]u = 0.


1 1 2
Using Eq. (5.36), we obtain
Li1 1t'< lr DWi!rc 11tial Equations
1
5.17
r,JoW, let v = u' . Th en , we
ha ve

ao (x )y ,v ' + 12 a ( x ) y +
0 1 a 1( x )y 1 Iv = 0
which is a first order equatio (5.37)
n in v.
separating the variables, we
obtain

\ntcgrnting. we obtain
~ = _ (2 ao Y1 + o, y 1) = _ l2 1
C10 Y1
y +
LYt
~J-
no

ln lv l= - 21 nl y, I -
J
(l 1(\'. )
, dx or v = -1-e -Jp <x> dx
ao (x ) ' yf
where p(x) = a1(x)/ao(x). Integ
solution is given by Y2(x) =
rating u' = v, we obtain u =
u(x)y1 (x). It can be verified
J
v( x) dx . The second linearly
independent
that the Wronskian of y . y
1 2 is equal to
W ( Yi , Y2 ) -_ e - J p ( x) dt -" Q
.,..
showing that y 1(x) and y (x)
2 are linearly independent.
Example 5.19 It is known tha
t 1/x is a solution of the differ
the second linearly indepen ential equation :c1-y" + 4.xy' +
dent solution and write the 2y = 0. Fi nd
general solution.
Solution Write y (x) = u(x )y
2 1(x) = u(x)lx. Here, p(x ) = a (x)
1 /a 0(x) = 4/x. Hence,
v( x) = l- e -J p(x )dx = xe 2 -J (41.t)d x
Y12 =x_2 ( - l4 ) =-..., l
.
X x-
u( x)

The general solution is y(x)


=
Jv( x) dx
J
= -dx2
x
l
= - -,
X
and Y2 (x) = u( x) y1 (.t.) = - l
2X -
= Ay,(x) + By2(x) = xA + B
x2 •

Exercise 5.2
Show that the given set of fun
ctions {y, (x), yi( x)} forms a ba
problem. sis of the equation and h~nc~ sol
ve the iruti1l "alue
1. ex, e 4 x, y" - 5y' + 4y =0, y(0) =2, y'(0) = 1.
2. e2x, e- 2x, y" - 4y =0, y(0) = l, y'(0) =4.
3. e- 3x, xe- 3X, y" + 6y' + 9y
= 0, y(0) = l, y'(0) = 2.
4. x 2, l/x2, x2 y" +x y' -4 y:
:0 ,y (l) =2 ,y '(l ) = 6.
S. x, x In x, x2y" - xy' + y =
0, y( l) = 3, y'( I) = 4.
n
Fi ua general solution of the
following differential equations
.
6. y" - 4y == 0.
1. y" - y' - 2y = o.
8. y" + y' _ y = O.
2 9. y" - 4y' - l2y = 0.
5.18 Engineering Mathematics
11. 4y" - 9y' + 2y :::: o.
10. y" + 4y' + y = o.
13. Y,, + 2/ + y :::: 0.
12. 4y" + Sy' - 5y = 0. ,, - 12/ + 4y:::: 0.
IS. 9y
14. y" + 2rcy' + rc 2y = 0.
17, 25y" - 20/ + 4y :::: 0.
16. 4y" + 4y' + y = o.
19, y" + 4y' + 5y :::: o.
18. y" + 25y = 0 .
21. (4D2 - 4D + 17)y:::: 0.
20. y" - 2y' + 2y = 0 .
23. (D2 + 9D)y :::: 0.
22. (D 2 - 6D + 18)y = 0.
24. [D 2 - 2aD + (a 2 + b2)]y = 0.
. h the following functions are solutions.
Find a differential equation of the form ay + Y
,, b , -o for w h1c
+ cy - •
x/4 e-( Jx)/4
25. 26• e '
27. 1'e- 2.r. 28. e2x, xe2x_
-Jix e3/x
29. e- x, xe-x. 30• e ' .
(S+3i)x, /S-Ji)x_
31. e-(n+ib)x, e-(a-ib)r. 32. e
Solve the following initial value problems.
33. y" - y = 0, y(0) =0, y'(0) =2.
34. y" - y' - 12y = 0, y(0) = 4, y'(0) =- 5.
35. y" + y' - 2y = 0, y(0) =0, y'(0) = 3.

36. d28
di 2 + g0 =0, g constant, 0(0) d0 (0) = 0 .
=a, constant, dt
37. y" - 4y' + 5y = 0, y(0) =2, y'(0) =- 1.
38. 25y,. - l0y' + 2y =0, y(0) = 1, y'(0) =0.
39. 4y"' + 12y' + 9y = 0, y(0) =- 1, y'(0) = 2 .
40. 9y"' + 6y' + y = 0, y(0) = 0, y'(0) = 1.
Solve the following boundary value problems.
41. y"' + 25y = 0, y(0) = 1, y(rc) =- 1.
42. y" - 36y = 0, y(0) =2, y(l/6) = lie.
43. y"' + 2y' + 2y =0, y(0) = 1, y(tr/2) =e-nn.
9y"' - 6y' + y = 0, y(l) = e , y(2) = 1.
113
44.
4S. y" - 4y' + 3y = 0, y(0) = 1, y(l) = 0.
46. Verify that (D - 2)(D + 3) sin x = (D + 3)(D - 2) sin x = (D2 + D _ 6) sin x .
47. Show lhat x 2Dy ~ D(x 2y).
48. find the conditions under which the following equations hold.
(i) (D + a)[D + b(x)]/(x) =- [D + b(x)][D + a]/(x), a constant.
(ii) [D + a(x))[D + b(x)]/(x) = [D + b(x))[D + a(x)]f(x).

Factorize the operator and find the solution of the following differential e . . . .
of order or by lhe direct melhod. quattons using the method of reduc uon

49. (D2 + 5D + 4)y =- 0. SO. (4D2 + 8D + 3)y = 0.


linear Differential Equations 5.19

51, (4D2 + 12D + 9)y = 0. 52. (D 2 + 6D + 9)y = 0.


53, (D2 - 4)y = 0. 54. (9D2 + 6D + l)y = 0.
55, Toe displacement x(t) of a particle is governed by the differential equation i + .i + bx = c.i, b > 0. For
what values of b and c is the motion of the particle oscillatory?
56, Find all non-trivial solutions of the boundary value problem
y" + w 2y =0, y(O) =0, y(,r) =0.
57, Find all the non-trivial solutions of the boundary value problem
y" + w 2 y =0, y'(0) =0, y'(1l') =0.
S8. Find all non-trivial solutions of the boundary value problem
y" + w2 y =0, y(O) =0, y'(n) =0.
S9. If a2 > 4b, then show that the solution of the differential, equation y" + ay' + by = 0 can be eJtpres.scd as
y(x) =ePx (A cosh qx + B sinh qx) where p =- a/2 and q = .Ja - 4b 12.
2

60. The motion of a damped mechanical system is governed by the linear differential equation my + cj, + k y = 0
in which m (mass), k(spring modulus), c (damping factor) are positive constants and dot denotes derivative
with respect to time t. Discuss the behaviour of the general solution when t ➔ in the following three
00

cases: (i) c2 > 4mk (over damping), (ii) c2 < 4mk (under damping), (iii) c2 = 4mk (critical damping).
In each case, obtain the solution subject to the initial conditions y(0) =.0, y(0) = u0 .
Find the solution of the following differential equations, if one of its solutions is known.
61. y" - y' - 6y =0, y 1 =e-ix. 62. y" + 3y' - 4y =0, y 1 =ex.
63. (r - l)y" - 2xy' + 2y = 0, YI = X, x 'i: ± 1.
64. ry" + xy' + (x2- l/4)y = 0, x > 0, y1 = x- 112 sin x.
6S. (x - 2)y" - xy' + 2y =0, x ,;: 2, y1 =ex.

5.3.4 Solution of Higher Order Homogeneous Linear Equations with Constant Coefficients
In this section, we shall extend the methods discussed in section 5.3.2, for the solution of higher order
linear homogeneous equations with constant coefficients.
Consider the nth order homogeneous linear equation with constant coefficients
aoy<n> + a1/n-l) + ... + an-ly' + any= 0. (5.38)

We attempt to find a solution of the form y = emx, as in the case of second order equations. Substituting
Y == e'"x, y<k> = mkemx, k = I. 2, ... , n in Eq. (5.38) and cancelling e'"x. we obtain the characteristic-
equation as
a m n + a1m n-1 + . .. + an-1m +a,.= 0.
0
(5.39)

The degree of this algebraic equation is same as the order of the differential equation. This equation
has n roots. All the roots may be real and distinct, all or some of the roots may be equal, all or some
of the roots may be complex. Consider the following cases.

Real and distinct roots


Let the polynomial equation (5.39) have all real and distinct roots us m1, IIIJ, . . . m,.. Then the 11
solutions
5.20 Engineering Math emat ics

mn X
m2x • . . y (x) = e (5.40)
Y1 ( x ) -- e "', x • y :!. (x) = e ., n
.
~re the linea rly indep ende nt solutions ?f the dtffe rent_. 1 t'on
1 (5.38 ). Sinc e :I- m 2 :I- • • • :I- mm
ia equa , • • •• Yn give n in Eq. (5.40 ) does not
m,
1t can be easil y show n that the Wronskian of the soluu
on_s Yt• Y2
vanis h and there fore they are linearly indep ende nt solutJ
OnS. . •s g,· ven by

Henc e, the set of the solut ions forms a basts d h
an l e ge neral so 1utton 1

= c 1e mix + c2e m2x+ · · · +ce


m nX
y(x) . (5.4 1)
n
Exam ple 5.20 Find the gene ral solution of the differenti
al equa tion
y"' - 2y" - 5y' + 6y = 0.
Solu tion Subs tituti ng y = emx, we obtain the characteristic equa tion as
m3 - 2m 2 - 5m + 6 = 0.
The roots of this equa tion are m = 1, - 2, 3. Since the
roots are real and disti nct, the gene ral solut ion
of the equa tion is given by

y(x) = Aex + Be- 2x + Ce 3x.


Exam ple 5.21 Solv e the differential equation y"' - y" - 4y' + 4y = 0.
Solu tion Subs tituti ng y = emx, we obtai n the chara cteri stic equa
tion as
3 2
m - m - 4m + 4 = 0 or (m - l)(m 2 - 4) = 0.
The roots of this equa tion are m = 1, - 2, 2 which are real and distin ct. The gene ral solu tion
equa tion is give n by of the

y(x) = Aex + Be-2x + Ce 2x.


Exa mpl e 5.22 Solv e the differential equa tion yiv - Sy"+
4y = o.
Solu tion Subs tituti ng y = e"'x, we obtai n the chara cteri
stic equa tion as.
4 2
m :-- 5m + 4 =0 or (m2 - ~)(m 2 - 1) = o.
The roots of this equa tion are m = - 1, 1, - 2, 2. The
gene ral solut ion is
y(x) = Ae-x +Be x+ Ce- 2x + De2x.

Exa mpl e 5.23 Solv e the diffe renti al equa tion 4yiv - 12y"
' _ y" + 27y' _ I Sy = 0.
Solu tion Subs tituti ng y = e"'x, we obtai n the chara cteri
stic equa tion as
4m4 - 12m3 - m2 + 27m - 18 = 0.
We find that m = 1 is a root. We write the equa tion
as
3 2
(m - 1)(4m - 8m - 9111 + 18) = 0, (m _
l)(m _ 2 )( 4 m~ _ ) = 0.
9
The roots of the chara cteri stic equa tion are m = 1, 2,
3/2, _ 312 Th
· 0
e oene ral solu tion is
y(x) = Aex + Be2x + ce-Jxn + De3xn.
Linea r Diffe renti al Equa tions 5.21
Example 5.24 Solv e the initial value problem

y'" - 6y" + l l y' - 6y = 0, y(0) = 0, y'(0) = - 4, y"(0)


=- 18.
SoIut•·on Subs tituti ng y - emx
- , we o b tam
· t he chara
cteristic equation as
3 2
m - . 6m + l lm - 6 = 0, or (m - l)(m - 2)(m - 3) = 0.
The roots of this equa tion are m = 1, 2, 3 and the gene
ral solution is
y(x) = Aex + Be 2 x + Ce 3 x.
Substituting the initia l cond ition s, we get

y(0) = 0 = A + B + C, y'(0) = - = A + 4B + 9C.


4 =A+ 2B + 3C, y"(0) =- 18
Solving, we obta in A= 1, B = 2 and C = - 3. Hence, the
-particular solution is y(x) =ex + 2e2:c - 3e 3x.
Real mult iple root s
The characteristic equa tion (5 .39) may have some mult
iple roots. Let r be the mult iplic ity of the root
m1, that is the root m = m 1 is repeated r times. Let the
remaining n - r roots be real and disti nct.
Substituting m = m 1 we obta in y (x) = em ix as one of
1 the solutions. We shall now show that the
remaining r- 1 linea rly inde pend ent solutions correspon
ding to the mult iple root m = m 1 are give n by
2
XY1 ,XY1 ,· • ·,X r-1 YI·
That is, the linea rly inde pend ent solutions in this case
are
e mix , xe mi x , x 2 e mix , . . . , x r-1 e mix
(5.42 )
since the Wro nskia n of these solutions W 0. *
If L [ y ] = aoy (n) + a1Y (n-1) + . • • + an-lY , + any
then, subs tituti ng y = emx in this equation, we get
L[em x] = [aom " + a1mn -l + . .. + an]e mx

= (m - m1rg(m) emx, g(m1 ) *0


(5 .43)
since m = m is a mult iple root of multiplicity r. Cons
1 ider now m as a parameter. Diffe renti ating
Eq. (5.43) with respe ct tom , we get

-1:_ L[em x] = r(m - mi)r -1 g(m) emx + (m - m 1)r dd [g(m


dm )emx ].
m
Now, Lis a linea r diffe renti able oper ator with respe ct
to the indep ende nt varia ble x. Sinc e m and
x are indep ende nt, we obta in

...E._ L [em x ] = L [...1:...... emx ] = L [xemx]


dm dm
= r (m - m Ir-1 g ( m) em X + ( Ill - m Ir d
d m [ g ( m) em x] .
(5.44 )
Since the right hand side of Eq. (5.44) vanishes at m = m1,
xe"q x is also a solut ion of the diffe renti al
equation. Diffe renti ating Eq. (5.44 ) with respect to m,
we get
5.22 Enginee ring Mathem atics

d ~gme
[ ( ) mx]
2 (5.45)
+(m- m1)' .
dm
. . . x2emix is also a solution . After
The nght hand side of Eq. (5 .45) vanishes at m = mi a~am: Henc~, d ( _ m )g(m)em x which
r - 1 differen tiations, the first term on the right hand side JS obtame as ' ·1 {" mix .1
. t·
vanishe s form= m 1• The other terms also vamsh . Th i e x'- e 1s a 1so a so 1u 10n.
form= m1. ere or ' h d 'd b
· .
If we differen tiate one more time, that is r times, the first term on the right an s1 e ecomes
. . , mix · not a solution . Hence, we find
r!g(m)e mx which does not vanish at m = m 1, showmg that x e IS .
that em1 x 'xem1 x 'x2em1 :c , . .. , x'-1em1x are the linearly indepen dent solution s corresp onding to
• ~he
h
multiple root m = m.1 . For example , if m = m is a multiple root of order 3, t en e m1x em1x and x2e ix
1 •x
are the linearly indepen dent solution s.
Examp le 5.25 Solve the differen tial equation y"' - 3y' - 2y = 0.
Solutio n Substitu ting y = emx, we obtain the characteristic equation as
m3 - 3m - 2 = 0, or (m + l)(m 2 - m - 2) = 0
or (m + 1)2 (m - 2) = 0, or m =- I, - 1, 2.
Corresp onding to the double root m = - 1, the linearly indepen dent solution s are e-x and xe-x. Hence,
the general solution is
y(x) = Ae 2x +(Bx+ C)e-x.
Examp le 5.26 Solve the differen tiable equation 8y"' - 12y" + 6y' - y = 0.
Solutio n Substitu ting y = emx, we obtain the characteristic equation as
8m 3 2
12m + 6m - 1 = 0, or (2m - 1)3 = 0, or m = 1/2, 1/2, 1/2.
-

The general solution is y(x) =(A+ Bx+ Cx 2)e x 12 •


Examp le 5.27 Solve the initial value problem
y"' + 3y" - 4y = 0, y(0) = 1, y'(0) = 0, y"(0) = 1/2.
Solutio n Substitu ting y = r, we obtain the characteristic equation as
m3 + 3m2 - 4 = 0, 2
or (m - l)((m + 4m + 4) = 0,
or (m _ l)(m + )2 = 0.
2
The roots of this equation are m = 1, - 2, - 2. The general solution is
y(x) = A e:c + (Bx + C)e-2:c.
Substitu ting in the initial conditio ns, we get
y(0) = 1 = A + C,
y'(x) = A ex + 8 e - l x -
2(Bx + C)e-h y'(O) _
' - 0 =A+ B - 2 C,
y"(x) = A e x - 4B e-l x + 4(Bx + C)e-2'-, y "(O) ::: l _
2 - A - 48 + 4C.
linear Differential Equations 5.23
• ·
olution of the system is A = 1/2' B -- 112 and C = 1/2. The particular solut10n
The s 1s
y(x) = [ex+ (x + l)e-2x] /2.

Simple complex roots


Since the co_eff~cients ~n ~he characteristic equation (5.39) are real, complex roots occur in conjugate
pairs. That IS, if P + iq is a root, then p - iq is also a root. In this case, the linearly independent
solutions are given by ePx cos qx and ePx sin qx. If the characteristic equation (5.39) has r complex
conjugate pairs of roots Pk± i q k• k = 1, 2, ... r, then the corresponding linearly independent solutions
pix X PI X • , P2X P2 X • p x d p,x .
are e cos q1 , e sm q1x, e cos q 2x, e sm q 2x, ... , e ' cos q,x an e sm q,x.

Example 5.28 Solve the differential equation yiv + Sy" + 4y = o.


Solution Substituting Y = emx, we obtain the characteristic equation as
4 2
m + 5m + 4 = 0, or (m2 + 4)(m2 + 1) = 0.
The roots are m = ± i, ± 2i. The general solution is
y(x) = A cos x + B sin x + C cos 2x + D sin 2x.
Example 5.29 Solve the initial value problem
yiv + 2y"' + lly" + 18y' + 18 = 0, y(0) = 2, y'(0)° = 3, y"(0) = - 11, y"'(0) = - 23.

Solution Substituting y = emx, we obtain the characteristic equation as


2 2
m4 + 2m3 + 1 lm2 + 18m + 18 = 0 or (m + 9)(m .+ 2m + 2) = 0.
The roots are m = ± 3i, - 1 ± i. The general solution is
y(x) = A cos 3x + B sin 3x + e-x (C cos x + D sin x).

Substituting in the initial conditions, we get


y(0) = 2 =A + C,
y'(x) = _ 3A sin 3x + 3B cos 3x + e-x (- C sin x + D cos x - C cos x - D sin x),

y'(O) = 3 = 3B + D - C,
y"(x) = _ 9A cos 3x - 9B sin 3x
+ e-x[- (C + D) cos x + (C - D) sin x + (C + D) sin x + (C - D) cos x]
_ sin 3x + 2e-x [C sin x - D cos x] .
= - 9A cos 3x 98
y"(0) = _ 11 = - 9A - 2D,
. os3x+2e-.r[Cc osx+Dsinx -C sinx+Dcosx}.
y"'(x) = 27A sm 3x - 278 c
y"'(0) = _ 23 = - 27 B + 2C + 2D.
. tions
Therefore, we have the system ot equa
A + C = 2, 38 - C + D = 3,
_ ?7 B + :.C + 20 = - ~3 .
- 9A - 2D = - 11 · -
. = C = l , D = I. The partic ular solution is
8 1' -{ ( . . )
The solution of this system 1s A = l.
. 3 . + sin )x + e . cos x + sm x .
y(x) = co~ .\ ·
5 -24
Eng inee ring Mat hem atic s
E .
xam ple 5.30 Fin d the non trivial solution s of the boundarY value problem
yiv _ m4y = 0, y(0) = 0, y"( 0) = 0, y(l) - 0 "(l) = o.
- 'Y . . .
Sol ut.·00 Ass ume the mx The cha ract eris tic equ atto n ts given by
solution to be of the form Y = e ·
m 4 - m 4 = 0 , or m 2 -- -+ m 2, or m = ± m, -+·zm.
The gen eral solution is given by
y(x) = A1ewx + B1e-wx + C cos mx + D sin
mx
= A cosh mx + B sinh mx + C cos mx + D
sin wx
Sub stitu ting in the initial conditions, we get

y(0) = A + C = 0.
y" = m 2 [A cosh mx + B sinh mx - C cos
mx - D sin mx] ;
y"(0 ) = m2(A - C) = 0, or' A - C = 0.
Sol vin g the two equations, we get A= O, C
= 0. We also have
y(l) = 0 = B sinh ml+ D sin ml, y"(l ) =
0 = B sinh ml - D sin wl.
Add ing , we obta in 2B sinh ml = 0, or B =
0. Therefore, we obtain D sin W l = 0. Sin ce,
*
non -tri vial solu tion s, we have D 0. Hence,
sin ml= 0 = sin nn, n = 1, 2, .. • •
we require
The refo re, w = n1tll, n = 1, 2, ...
The solu tion of the boundary value problem
is
yn(x) = Dn sin (nn xll) , n = 1, 2, .. . ,
By sup erp osit ion principle, the solution can
be written as

y(x )
-
=~ Dn sin (nn xll) .
n=l

Mu ltip le com ple x roots


Thi s cas e is a com bin atio n of the two earlier
cases of real multiple roots and sim ple com
Now , if p + iq is a mul tipl e root of order m, plex roots.
then p - iq is also a multiple roo t of ord er m.
if Pi+ iq 1 is a dou ble roo t, then p 1 - iq1 is also For exa mpl e,
a double root. The corr esp ond ing line arly inde
solu tion s are pen den t

e PIX cos q 1x, e PIX sm



q1x , xe PIX cos q 1x, xe PIX sin q x.
1
Exa mp le 5.31 Sol ve the differential equation yiv + 32y " +
256y = o.
Sol utio n Sub stitu ting y = emx , we obtain
the characteristic equation as
4
m + 32m 2 + 256 = 0, or (m2 + 16)2 = 0.
The roo ts of this equ atio n are the double root
s m = ± 4i. Therefore, the general ·solu tion
is
y(x) =(A x+ B) cos 4x + (Cx + D) sin 4x.
Line ar Differential Equations 5.25
giercise 5.3
find the !~neral, ~lut ion of the following differenti
al equations .
1. Y - 9Y - o. 2. 2y'" + y" - 13y' + 6y = 0.
3, 3y,,, - 2"
y - 3' y + 2y = O. 4, y1v - l 3y" + 36y = 0.
5, 4yiv - 12y"' + 7y" + 3y' - 2y = 0.
6, ylv + y"' - 4y" - 4y' = 0.
7, siv - 6y"' - 7y" + 6y' - y = 0.
8. )44y 1v - 25y" + y = 0.
9, y"' - 2y" + y' = o.
10. y"' +4y" +Sy' +2y = 0.
11. y"' - 2y" - 4y' + Sy = 0 .
12. 27y'" - 27y" + 9y' - y = 0.
13, iv - 1ly"' + 35y" - 25y' = 0.
14, iv - 3y"' + 3y'' - y' = 0.
1s. 4l + 4y"' - 3y" - 2y' + y = o.
16. 9y1v - 66y' " + 157y'' - )32y ' + 36y = 0.
17, y'" + y' = 0.
18. y"' - 2y". + 4y' - Sy = 0.
19. y"' + 5y" + Sy' + 6y = 0.
20. y"' - 1y" + 19y' - 13y = 0.
21. l + Sy" - 9y = 0.
22. iv+ y"' + l4y" + 16y' - 32y = 0.
23. 4yiv + lOly" + 25y = 0.
24. yiv + 2y"' - 9y'' - 10y' + 50y = 0.
25. iv + s9y'' + 625y = 0.
26, iv + 2y" + Y = 0.
Find a homogeneous linear differential equation with
real constant coefficients of lowest order wruch has
following particular solution. the
27. 5 +ex + 2e3x. 28. e-x + cos Sx + 3 sin Sx.
29. xe-x + eix. 30. 1 + x + ex - 3e 3x.
2
31. x e2x + 2e- x.2
32. 3 cos 2x + 5 sinh 3x.
Solve the following initial value problems.
33. y"' - 2y" - Sy' + 6y =0, y(O) =0, y'(O) =0, y"(O
) = 1.
34. 4y"' - 4y" - 9y' + 9y = 0, y(O) = 1, y'(O) =0, y"(O
) = 0.
35. y"' - Sy"+ 1y' - 3y = 0, y(O) = I, y'(O) = 0, y"(O
) = - 5.
36. iv - 2y"' - 3y" + 4y' + 4y = 0, y(0) = 3, Y' (0) = 3 "(0) 3 y'"(O) = 6.
,Y = ,
37. iv+ y" = O, y(O) = 1, y'(O) =2, y"(O) =- 1, y"'(O
) = - 1.
38. y"' - y" + 4y' - 4y =0, y(O) =0, y'(O) = 3, y"(O
) = - 5.
39. y'" + y" _ 2y = O, y(O) = 2, y'(O) = 2, y"(O) = -
3.
40. iv_ 3y"' = o, y(O) = 2, y'(O) = 5, y"(O) =15, y"'(O
) =27 .
Find the solution of the following differential equa
tions satisfying the given conditions.
41. y"' + 1e2y, = o, y(O) = o, y(l) = 0, y'(0) + y'(l)
= 0.
42. y"' - 36y' = 0, y(0) = 2, y'(0) = 12, y'(l) = 6 sinh
(6) + 12 cosh (6).
43. -l + 13y" + 36y =0, y(0) =0, y"(0 ) =0, y(rr/
2) =- l. y'(tr/2) = - 4.
44. yiv - w4y = 0 w ~ 0, y(0) = 0, y''(0 ) = 0, y(rr) = 0,
y"(,r) - 0.
45 iv =
'
• Y + 1Oy" + 9y = 0, y '(O) _ 0
- ,
y'"(O
'
=
) O y'(rr/2) 5, y'"(, r/2) = - 53 ·

S.4 Solution of Non-Homogeneous Linear Equatio


ns . .
.. 1 the ,,eneral and parti cula r so\ut1ons
In th · t ct·ng
e previous section, we have d1scussed meth od·s for m · ~
5.26 Engineering Mathematics

shall discuss methods


for finding the general
quations. In this section, we
Solution
Ouuon of aa non
non-homogeneous of the
linear equation (see Eq. (5.1)) 1o
(5.46)
LDI=apla)y(x) +aj)y) +.. .
+ay-1t)y' +a,)y = r(), aol)*
1s known. We
nen the general solution of the corresponding homogeneous linear equation L [vl =0
present the following theorem.
Theorem 5.5 If {yi(x), yat), is a basis and cy () + C2y2lx) +
is the
.

.ynlx))
..
general = 00and
.+ Cayalt) .

O n of the corresponding homogeneous linear equation L )


particular and if y,(x) is any
if yp(x) any particular
n a solution not containing any arbitrary constants) of the non-homogeneous equation (5.46),
then the general solution of equation (5.46) is given by
yr)= C1 yilt) + C2y2(x) + . . .+ Cn ynX) + YpX). (5.47)
Proof Since y,(x) is a
particular solution, we have

Lyp(x)) = a0y" + a y " - + . . . + ag-1y, + dnyp = r(x). (5.48)


Subtracting Eq. (5.48) from (5.46), we obtain
ao (y - y") + a (yn-1)- y - ) ) +.. . + an-i(y'-y) +a, (y - Yp) = 0.
(5.49)
Denote y -

yp =
z. Then, from Eq. (5.49) we obtain
aoz+ az-)+.. .+ an-1Z + a,Z =0. (5.50)
But, this equation is the corresponding homogeneous equation of Eq. (5.46), whose basiIS 1S
y1(), y2*), y,(x)). Hence, the general solution
. .
..
of Eq. (5.50) is given by
z C1 y1(x) + C2 y2() +
=
. . . + Cn YnlX).
Replacing z = y -Yp» and taking y, to the right hand side, we obtain

yx) =
C1yi(x) + C2y2(x) + . .+Cny,x) +Yp(). .

(5.51)
Since, this solution contains n arbitrary constants, it is the general solution of the Eq. (5.46).
From the above theorem, we conclude that the solution of
of the sum of the following two parts.
a
non-homogeneous equation consists

i) The general solution of the corresponding homogeneous


equation. This solution is called the
complementary function and is denoted by ye(x).
ii) A particular solutionof the non-homogeneous equation. This solution
is also called
integral of the non-homogeneous equation and is denoted by y, (x).
a particular
The general solution of the non-homogeneous equation is then written as

yx) =
ye(o) +
yp(x).
Now, suppose that the right hand side r() is the sum of a number of functions
r() =r1) + r2(x) +
+Tm) (5.52)
Let va(x), i =
1, 2,.. .
, m be any particular solutions, not containing any arbitrary constants, of the
equationss
ay4 ay) + . .

.+an-1y +an y ri (x), i =


1, 2,.
=
. .
, m.
(5.53)
Linear Differential Equations 5.27

Then, Ypi * p2 ***.


*Ypm 1S the particular integral of the equation
(n)
a0y+ay +. .+ an-1y + a,, y= ri(x) + r2(x) + . +
TmX) r . .

and hence of the given non-homogencous linear


ver In other
i.
cquation. This can be proved by summing Eq. (5.53)
words, if the right hand side of
ove
Eq. (5.46) consists of sum of a number of functions,
then particular integrals of the Eq. (5.53) can be obtained with
the particular integral of Eq. (5.46) is then
respect to each of the functions and
given by the sum of these particular integrals.
The methods
for finding y, (x) have been
discussed in the previous section. In the
of this section, we shall derive methods for remaining part
finding the particular integral of the y,(x) non-homogeneous
equation.

5.4.1 Method of Variation of Parameters

Consider the second order


non-homogeneous linear cquation
aa)y" + a,(t)y' + a2(t)y = r (x), aolx) # 0.
(5.54)
We shall discuss general method of solution, called the method of variation of
a

can always be used to find parameters, which


a
particular integral whenever the complementary function of the equation
is known. Consider first, the solution of the
corresponding homogeneous equation
aox)y"+ aj«)y + a2(r)y = 0, aox) # 0. (5.55)
Using the methods given in the previous section, we can find two linearly independent solutions yi(r)
and y(t) of the equation (5.55). The complementary function is given by
y ) = Ay1) + By2()
(5.56)
where A and B are arbitrary constants. The idea behind the method of variation of
parameters is to vary
the parameters A and B. That is, we assume A and B to be functions ofx and determine A(x), B(x) such that

y)=AW)ynl) + B()y2x) (5.57)


is thegeneral solution of Eq. (5.54). Now, y) contains two functions A(r) and B(:) which are to be
determined. Therefore, we need two equations to deterimine them. One equation is obtained by
substiruting ylx) from Eq. (5.57) in Eq. (5.54). The determination of the second equation is at our
disposal. This equation is chosen such that the determination of A(r) and B(r) is simple. Differentiating
Eq. (5.57), we obtain
y(x) = A'y1 + Ayí+ By% + B'y; = (A'y1 + B'y2) + (Ayí + By;). (5.58)
If we differentiate this equation again, then the equation would contain the second derivatives A" and
Bof the unknown functions. In order that these derivatives are not used, we set in Eq. (5.58)

A'y + B'y2 = 0. (5.59)


which gives us the second equation to determine A() and B(v). Now, ditferentiating y'() = Ayí + By
we obtain

y"(x) = Ayf+ A'yí+ By{+ B'y$. (5.60)

Substituting the expressions for ylx), y'tr) and y") in Eq. (5.54), we obtain

a0(x)[Ayí+ A'yi+ By2+ B'y%] + aj(*)[Ayí+ By21 + az(*)\Ayi + By2l r(x)


=
5.28 Engineering Mathematics

ai(x)yí +az(x)y1
or
ao (x)[A'yí+B'y2] + A[ao(x)y+
+B[ag(x)yi+ aa(x)y2 + a2(x)y2]
= r(x).
we obtain
Si
once, y() and y2(a) are the solutions of the homogeneous equation (5.55),

A'yí+ B'y2 = 8(x). (5.61)


a(x)[A yí + B'y2] =
r(x), or

Since ao(r) * 0 on the given interval I, g(x) is continuous on 1. Solving the equations

A'n +B'y2 = 0

A'yi+ B'y%= g(x),


we obtain

A' . &(x)y2 B'=


8*)y
(5.62)
y-y2 y y12-Y2yí
We note that the Wronskian Wo1. y) is

W= 2 |.
yiy*y1ys - y2í *0
since y, y2 are the linearly independent solutions of the homogeneous equation. Hence, we can write
Eqs. (5.62) as

8(X)y2 8&)y,
A'=- and B'=
(5.63)
Wx) Wx)
Integrating, we obtain

A(x) = - 8(x)y2(x)
dxd+Cj and B(x) = | W(x) dx +C2
W(x) 5.64)
Substituting in Eq. (5.57), we obtain the general solution which contains two arbitrary constants. If
we do not add the arbitrary constants while carrying out integrations of Eqs. (5.63), then we obtain
the particular solution as yp) =A)Y1) + BT)Y20), which does not contain any arbitrary constants.
then given by ylx) y ) + yx).
The general solution is
=

The method is applicable both for constant coefticient and variable coefficient problems. The
method can also be easily extendea to cquatons or any order. At each differentiation step, we set the

Dart containing the derivatives of the unknown Tunctions to zero, until we arrive at the final substitution
consider the third order equation
step. For example,
aox)y" + aj)y" + az*)y' + astr)y = r(v), aglr) * 0.
(5.65)
function is
The complementary
yx) = Ayit) + By;(a) + Cys(x)

where y. Y2» Ys are the linearly independent solutions of the corresponding homogeneous equation
arbitrary constants. We assume the solution as
and A, B, C are

yx) = A(x)yi (x) + B(r) y2(r) + C()yi(r).


(5.66)
Linear Differential Equations 5.29

Following the procedure discussed earlier, we obtain the required equations for determining A().
Bx) and Clx) as

A'C) y + B' (x)y2 +


C'(x)y3 = 0

A'(x)yi+ B'(x)y2 + C'(x)y} =0


and A'(x)y+ B' (x)y{+ C'(x)y= =
g(x). (5.67)
The determinant of the coefficient matrix is the Wronskian
Wy Y2 Ya) #0. We determine A(x), B(X),
Clx) and substitute in Eq. (5.66) to obtain the general solution.
Example 5.32 Find the general solution of the equation y"+3y' +2y = 2e, using the method or
variation of parameters.

Solution The corresponding homogeneous equation is y"+3y


is m3m +2 0 and its roots are m
=
+2y=0. The characteristic equation
1, -2. Hence, the= -

is
complementary function
Yea) = Ay1() + By2(x) = Ae + Be-2x

where y)=eand y2x) =


eA are two linearly independent solutions of the homogeneous equation.
Assume the general solution as

yx) = A()e + B(x)e*

We have g(r)r(x)lag(x) 2e".


= =

The Wronskian of y1(«), yzlx) is given by

e-2x

-2e-2x -
Using Eq. (5.64), we obtain the solutions for A() and B(x) as
8(r)ya(x) dx
Ax)= - W q-2 d x cj= c + e4x +

g(x)y1il)
W
dx +
2 edx+ca= -ge+ C2.
The general solution is
) = A(«)e+ B(t)e

= (e" + C)e + e+c2le*=ee +c2e*


Example 5.33 Find the general solution ofthe equation y+ 16y= 32 sec 2x, using the method of
variation of parameters.
Solution The characteristic equation of the corresponding homogeneous equation is m+16 = 0.
he characteristic roots are m = t 4i. The complementary function is given by

By2(r) =A cos 4x + B sin 4r


yx) =
Ay) +
5.30 Engineering Mathematics

solutions of the homogeneous


cos 4x and y(x) two linearly independent
sin 4x
cquation.
=
By
= are
the method of the variation of parameters, we write the genea

yx) =
A(x) cos 4x + B(x) sin 4x.
We have s r ) = r(x)/ag(t) = 32 sec 2x. The Wronskian of yi. y2 is given by

cos 4x sin 4 4.
W()= 4sin 4x 4 cos 4x

Therefore, from Eq. (5.64), we obtain


A(x) =. &(x)y (x)dx +C1 =-32 sec22x sin 4x dx + C1
W

-16 sin 2x dx + c = 8 cos 2x + C1

B(x)
B(x) =| sx)yi) ax +eq = 32 sec 2x cos 4x dx +C2
W

2 cOSCOsx2x dx + ca 8 (2 cos 2x -

sec 2x) dx + C2

= 8 sin 2x 4 In | sec 2x +tan 2x | +


C2
-

The general solution is


yr) = A(x) cos 4x + B(x) sin 4x

=
C1 Cos 4x + C2 Sin 4x + 8 cos 2x cos 4x + 8 sin 2x sin 4x
-4 sin 4x In | sec 2x + tan 2x|
C Cos 4x + C2 sin 4x + 8
2x + tan 2x 1. cos 2x -

4 sin 4x In | sec
Example 5.34 Find the general solution of the equation y" 6y"+ 11y' - 6y = e .

Solution The characteristic equation of the


m- 6m+ 11m -6 0 and its roots are m
corresponding homogeneous equation is
1, 2, 3. The =

complementary function is given by


yx) =
Ae"+ Be 2 +Ce3x
By the method of variation of parameters, we assume the solution as

y(x)= A(x)e* + B(x)e2" + C(x)ex,


We have &(r) =
r(«/ag(r) e", =

From Eqs. (5.67), the equations for


determining A(P), B(r) and C(r) are
A'e+ B'e" +C'e3 =0
A'e +2B'e+ 3C'e = 0
A'e 4B'e +9C'eS = e
The Wronskian of yi =e* y2 = e" ys= e" is given by
5.31
LinearDiferential Equations

ex e2x e3x
111
W(x) =|e 2e2x
3e=1 2 32e
ex 4 4e2x 9e3 1 49
By the Cramer's rule, we obtain

0 e2x e d

WA'=0 2e2x 3e3 | =e", or A'= e",


4e2x 9e3x

Integrating, we get A = - .

-23C1
Similarly, we have

0 e
WB'= |e 0 3e3x
3e=-2e3", or B'= -2e= -e-3s
e 9e 2e

ee2
WC'=e 2e2x 0 , or C ,
4 e 2r
e
Integrating, we obtain B(x) = e c a andC(x) = -ed# + Cy. The general solution is

y(x) = A(x)e* + B(x) e" + C(x)e3

cje+ C2e"+ Cze*-


and y2 I/x are two linearly independent solutions of the
Example 5.35 It is given that y
=
= x
associated homogeneous equation of x*y" +xy-y= X, X *0. Find a particular integral and the

general solution of the equation.


of parameters, we write
Olution By the method of variation
y(x) = A(x)x + B(x)

ne Wronskian of yi(t) =x and y2(x) = I/x is given by


1/x
W-
-1/x
obtain
We have l/x. Using Eq. (5.64), we
g(x) =
rxlapx) =
S.32 Engineering Mathematics

A)- --iInlz|+o

B(x)=S()y,(x)
W
The particular integral is

x)=Acr)x +B( = In |x|-4


The general solutionis
+ca+ n |x| -

r)
=»e(a) + y)= cx
or x) =ejxe+ n|«|, where e=e-

Exercise 5.4
Find the general solution of the following differential equations, using the method of variation of parameters.
1. y"-2y - 3y = e. 2. -4y'+ 4y =
e
3. y"+ 4y =cos x. 4. y+y = sec x.
5. y + y =cosec x. 6. y+y = tan x.
7. y - 4y +3y = e' 8. + 4 y = sec 2x.
9. y+ 4y = cos 2x.
10. +4y+4y= e sin x.
11. y"+6y' +9y = e"/x. 12. y"+2y' + 2y = e cos x.
In the following problems, using the method of variation of parameters and the given linearly independent
solutions, find a particular
integral and the general solution.
13. *+ r-y=x. yi =x, y2= 1/x. 14.
+xy-4y= * In |x|.yj =ry= l/x
15. "-xy +y l/x', yi =x, y2 =x ln |x|
=

16. y-2xy+2y =r+x, y1 =x, y2 =*.


17. y+ 4y' + 8y =16 e" cosec 2x, y e" cos 2x, y2= e sin 2x.
=

18. y"+4y sec 2x, yj 1, y2 cos 2x, y3 sin 2x.


= = = =

19. -6y"+ 12y' 8y= "lx, y ey= xe, y=fa


-
=

20. Show that the general solution


can always be written as
of the equation y" + k°y= 8(), where k * 0 and g(t) is continuous on l.

y(x) =A cos kx + Bsin kx sin k(x -

t)g(t)dt.
5.4.2 Method of Undetermined Coefficients
In the previous section, havee discussed the
we
method of
solution of the differential equation variation of
paranmeters for finding the
a
y+ay-+..+dn-1Y +an y =r(x)
Linear Differential Equations 5.33
where ao d1, an are constants. In
the cases when the right hand side r(x) is
containing exponenuals, polynomials, cosine and sine
of a spec1al rorm
then the particular integral can be easily obtained by the methodsums functions,
or products of these
functions,
basic idea behind this approach is as follows. of undetermined coefficients.ne

If r(x) is of exponential form


e"A, then its derivatives also
ifr (x) pe
=
p constant, then we can contain exponentials e"" only, that 1s,
choose the particular
determinecby substituting ypT) in the given equation and integral as y, (x) ce"", c constant ana =

is, the equation is identically satisfied. comparing both sides of the equation. i na
If r() is a cosine or a sine
function, cos mx or sin mx, then their
cos mx and sin mx. In other
words, if r (x) p cos mx or p sin =
derivatives contain the terms
particular integral as yp(x) c1 cos mx + C2 Sin mx. The constants p constant, then we can choose the
= mx,

y,x) in the given equation and comparing both sides of c1, C2 are determined by
the equation. substituting
If r(x) is of the form x", then
its derivatives contain
r(x) = px".p constant then we can the terms x", " , , x, 1. Hence, when
choose the particular as integral
Ypx) COx" +Cx"+.=
.+
Cm-1x + Cm .

(5.68)
where Co» C1 , Cm are constants.
If r(x) is of the forms
sin bx. Hence, when
e" cos bx or e" sin bx then their derivatives contain the
terms e cos bx and
r(x) e" cos bx or e" sin bx, then we
=
can choose the
particular integral as
yp(x) e" (C1
cos bx + c2 sin
=

bx).
However, if any term in the (5.69)
choice of the particular
homogeneous equation, that is, a term in the integral is also a solution of the
complementary
function, then we multiply this term by
corresponding
x or
by x" (if the term in the
complementary
m). If r(x) is the sum of a number of function corresponds to a
functions, then the contribution withmultiple to
root of
is included in the multiplicity
choice of the particular respect each of the terms
integral.
Example 5.36 Using the method of undetermined
the differential equation y" + y= 32x'. coefficients find the general solution of
Solution The characteristic
m=ti. The equation of the homogeneous equation is m* + l = 0 and its roots
Since r(x)
complementary function is yc(x) A cos x +B sin x. = are
=
32x3, we choose the particular integral as
y,x) C1x + C2* + C3x + C4 =

Substituting in the given equation, we get


(6cx+2c2) +(C1xr +C2x+ C3x + Ca) = 32x3.
Lomparing the coefficients of various powers of x, we get
3 2 , C2= 0, 6c +C3 = 0, 2c2 + C= 0.

ne solution of the system


The is c =
32, c2 =
0, C3 -

192, c4= 0.
general solution is Therefore, ypCr) =
32r- 192.
yx) = A cos x + B sin x + 32x (r2- 6).

Example 5.37
*Ample 5.37 Find the general solution of the difterential equation y- 2y' -

3y =
6e"- Se
5.34 Engineering Mathematics

SOuton The characteristic equation of the homogeneous equation is m -2m -5 =0 and its roots

are m = -1,3.
The complementary function is ylt) = Ae" + Be*
root 1) and the right hand
note that e appears both as a term in y(r) (due to the simple
nm= -

SIde r(). The term e' appears only in r(x). Hence, we choose the particular
side integral as

yp()= cxe"+ cze.


We have (1 x)e"*+ cje'.y(x) -i(2 -

x)e""+ c2e*,
y,(r) = z
ci -

Substituting in the given equation, we get


Cil-(2-x) -

2(1 -x) -3x]e" + -2-3Je" =6x - 8e*


ca[l
or
-4ce-4c2e" 6e 8e. = -

Comparing the coefficients of e ande, we get c = - 3/2, c2 = 2. The general solution is

y(x)=Ae+Be' xe"+ 2e. -

Example 5.38 Find the general solution of the equation y"+9y = cos 3x.
Solution The characteristic equation of the homogeneous equation is m+9 =0 and its roots are
m t 3i. The complementary function is

yr) = A cos 3x + B sin 3x.

We note that cos 3r appears as a term in y.x) and the right hand side r(x). Hence, we choose the
particular integral as
plr)= x(¢i cos 3x + c2 sin 3x).
We have y(x) =
cj cos 3x + C2 sin 3x +3x(-c1 sin 3x + C2 cos 3x)
yx) =
6(- ci sin 3x + C2 Cos 3x) + 9x(-c1 cos 3x -

c2 sin 3x).
Substituting in the given equation, we get

y+9y sin 3x [- 6c - 9x¢2 + 9xcal + cos ax [6c- 9xc/ +9xc] = cos 3x

or
-

6c sin 3x + 6c2 cos 3x =


cos 3x.

Comparing both sides, we get ci = 0 and c2 1/6. The


particular integral is y,(x) =
(r sin ax/6.
The general solution is

yx) = A cos 3x+ B sin 3x + x sin 3x.

Example 5.39 Find the general solutuon of the equation y" +4y'+4y = 12-

Solution The characteristic cquation or ine


nomogeneous equation is m 4m
and its roots are m = -

2, -2.
+ 4 = (m +2)= 0

The complementary
function is y,(r) =
(Ar+ 8)e-2

...a that 2 and xe" are present in the


complementary function (due to the double root
Linear Differential Equations 5.35

m
2) and eA is also a term on the right hand side r(x). Therefore. we choose the particular
as
integra
Ypx) = cjxe-2

We have yp(x) =
cil2x -

2x°]e, yf(x) =
ci[2 -
8x +
4x]e
Substituting in the given equation, we get

p+4y + 4yp= C1[(2 8x + 4x2) + 4(2x - 2x2) + 4x2 Je-2x = 12e-2*

2c1ex= 12e-2x
Comparing both sides, we get C =6. Therefore, the particular integral is y,(x) = 6x*e. The general
solution is

y(x) =
(Ax+ B)e* + 6xe =
(Ax + B + 6xjea,
Example 5.40 Find the general solution of the equation y"- 4y'+ 13y =
12e sin 3x.
Solution The characteristic equation of the homogeneous equation is m- 4m+ 13 = 0. The roots
of this equation are

m=
4 t 16-52 -2 t 3i.
2
The complementary function is yx) =
e*A cos 3x + B sin 3x).
We note that e" sin 3x appears both in the
complementary function and the right hand side r(x).
Therefore, we choose
yx) = xe"(c1 cos 3x + c2 sin 3x).

We have

yCx) = (1 +2x)e (e1 cos 3x + c2 sin 3x) + 3 x e - c sin 3x + oa Cos 3)

yx) = (4 + 4x)e (ci cos 3x +c2 sin 3x)

+6(12x)e(-C1 sin 3x + c2 cos 3x) + 9xe** (-Ci cos 3x - c2 sin 3x).


Substituting in the given equation, we get
-4yh + 13yp = e cos 3x [c(4 + 4x) + 6cz(l + 2x) - 9cx - 4ci(1 + 21)

12xc2 + 13cx] + e* sin 3x lczl4 + 4x) -

6c(1 + 2x)- 9cx


- 4c2(1+ 2x) + 12cx + 13xc2] = 12e sin 3x

r
6czecos 3x - 6ce sin 3x = 12 e sin 3.

Comparing both sides, we get ci 2 and c 0. Therefore, the particular integral is


px)= - 2xe cos 3x. The general solution is

e [A cos 3r + B sin 3r -

2x cos 3r].
yx) =

Example 5.41 Find the general solution of the differential equation y"- 2" - 5y' +6y = 18e*.
5.36 Engineering Mathematics
is
Solution The characteristic equation of the homogeneous equation
0, orm =
1, -

2, 3.
(m 1)(m + 2)(m -

3) =

m-2m-Sm + 6 = -

The complementary function is y.(x) = Ae* + Be+Ce".

Choose the particular integral as y,(x) = C1xe".

x)e",y"=c\(3 + x)e*.
We have yp=C(1 +
x)e".y=c (2 +

Substituting in the given equation, we get

x) -2(2+x) 5(1 + x) + 6x]


=ce"[(3
-

- 2 y - 5y + 6yp +

= - 6c^e = 18e.
Comparing both sides, we get c= - 3. Hence, the particular integral is y, = -3xe". The general

solution is

yx) =Ae*+ Be + Ce3 3xe".


Example 5.42 Find the general solution of the differential equation

y-6y 12y -

8y = 12e*+ 27e.
Solution The characteristic equation of the homogeneous equation is
m-6m+ 12m -8 = (m -2) = 0, orm =2, 2, 2.
The complementary function is y,) = (Ax + Bx +C)e*. Note that m = 2 is a triple root and e is
contained in a term in rl). Therefore, we choose the particular integral as
yp(t) = C1x'e + c2e"

We have y= c\3x*+2x*)ex c2e"*, y= ci(6x + 12x+ 4x3)e2* + C2e*,


-

y C(6 + 36x + 36x2+ 8x3)e2-czex.


Substituting in the given equation, we get

6y+ 12y -8yp =Ce*[(6 +36x + 36x* + 8x*) 6(6x + 12x2 + 4x3)
-

+ 12(3 +2x)- 8x]+ 2e-1-6-


12 8]
6ce-27c2 e=12e+27e
Comparing both sides, we get Ci 2 and c2 l.
=
=

Therefore, the particular integral


-

2xe2- e", The is


Y,x) =
general solution is

yx) (Ax+ Bx +C)e"+2xe-


=

e
Exercise 5.5
aeneral solution of the following ditierential
equations by the method
1. - 3 y - 10y = I +
2. 2y-y-
of undetemined coefficients.
3y =x+x+ 1.
Linear Differential Equations 5.37

4y-y
=e"+ e 4. 3y+2y' -y =
e+x
3. 6. y+ 4y' +3y =6e
+6y+8y = er"+e
5. 8. y-y-6y =Se + 10«3.
-2y Sea+e",
=

1. 2y"+3y
5y-2y 14e/3 10. y"+3y' + 2y = cos x + sinnx.
9. 3y+ 12. y+ 4y' - 5y = 34 cos 2r -2 sin 2x.
= 39 cos 3x.
+y-6y
11. 14. y+ 16y = 16 sin 4x.
sin 5x.
13. +25y=
50cos Sr +30
16. 4y-4y'+ y = 6 e ,
8e* +
15. y-4y +4y
=

+6y+9y 26e +
5e2 =
18. y+y = e" sin .
17.
3x. 20. -4y' + 5y = 16e cos x
19.y+2y +10y = e sin
6e"" sin x cos x.
22. y+4y+4y = 6e* cos x.
y-6y +13y
=

21.
24. y-4y' +3y = 4 cosh 3x.
23. +3y+2y 12e sin x. =

26. y"+3y"-4y 12e+ 9e.


25. y"+4y"-y-4y 18e =

28. y"-"+y' -y= 6 cos 2x.


"-9y"+ 27y' 27y 36er
-

27.
29. -2y"+ 4y' -8y 8(+ cos 2).
128 4x. 31. y -y =r'+ 1.
30. y-256y = cos

33. -3y"-4y = 60e


32. y+3y" + 3y" + y =2x+4.
34. y+6y" + 12y" + 8y' = 60e 35. y - 16y" = 8x + 16.

54.3 Solution of Euler-Cauchy Equation


constant coefficient
discussed methods for finding the solution of the
In the previous sections, we have coefficient
not exist, in general, for the variable
differential equations. Closed form solutions do
linear equations. However, for the Euler-Cauchy equation
x*0 (5.70)
agy + dxly)+.. .+a,-1xy' +a,y =r(x),
be obtained by using one of the
where ao» d1 , an are constants, closed form solutions can

following two procedures.


We shall illustrate these procedures using the second order equation

ag*y +ajXy +a2y = r(t), ag *0, x * 0. (5.71)

Consider first, the corresponding homogeneous equation


agxy + a1xy + azy = 0. 5.72)

find a solution of the form y = x", We have y' mx"-


= and y" =
m(m -

1)x"
We attempt to
Substituting in Eq. (5.72), we get

a m(m -

1) + am + az]x" = 0. (5.73)

Cancelling x", we get


ag m(m
-

1) + am +a2 =
4g m + (a1 -

ag)m + a2 = 0 (5.74)
Which is called the auxiliary equation corresponding to the Eq. (5.72). Equation (5.74) has two roots
In these cases,
m, m2, which may be real and distinct, real and equal or complex conjugates.
we

obtain the following solutions.


5.38 Engineering Mathematics
Real and distinct
roots
t the roots m and m2 are real and distinct, then the two linearly independent solutions are

y1 (x) x " = and y2(r) = x"2, (5.75)


The general solution is given by
y(x) =A x + Bx"2 (5.76)
where A., B are
arbitrary constants.
Example S.43 Find the solution of the differential cquation x*y"+ 2xy - 4y =

Solution Here,.o1, aj = 2 and a = - 2. The auxiliary equation is


o m(m- 1)+ a m + az = m2 + m - 2 = 0, or (m + 2)(m - 1) = 0.

h e roots of this equation are m = 1, - 2 . Hence, the two linearly independent solutions are

1(x) = x, and y2(r) = x


The general solution is
y(x) Ax + (B/x*). =

Example 5.44 Find the solution of the differential equation 2x*y"+xy -6y = 0.
Solution Here, ag 2, a 1, and az = 6. The
= -

auxiliary equation is

m - 1 ) + am + a = 2m - m - 6 = 0, or (m -2)(2m+ 3) = 0.
The roots of this
equation are m =
2, -

3/2. The two linearly independent solutions are

y1) =
x*, and y2lr) =
3
The general solution is y(x) =Ax2 +
Real and equal roots
Let the roots of the auxiliary equation be real and
equal, that is, m =m^ is a double root. Then
m (a0 a1)/(2a). Since, the discriminant of Eq. (5.74) vanishes in this case, we can
-

also write
m a2/ao (product of roots). Then, yi(x) =x"i is one of the linearly independent
=

second linearly independent solution can now be obtained solutions. The


by the method of reduction of order (see
section 5.3.3). Write ya(x) = ulx)y1). We have

y uyi+ u'y1. y= uyí+ 2u1yí+ u"y1.


Substituting in Eq. (5.72) and simplifying, we get

agx (uy+2u'yí+ u"y1) + ajx(uyí + u'y) +azuy1 0


or aoYx2u"+ xu'(2d0ryí+ a1y1) + u{a0x *yi+ axyí+a2y1) 0. =
(5.77)
Since y(x) is a solution of Eq. (5.72), the third term Eq. (S.77) vanishes. in
where m = (a0 - a/(2ag), we obtain
Further, since yi(r) = x"
Linear Differential Equations 5.39

2agXyí + a1y1= (2agm + a)x"' =


agx" =
a0y1.
herefore, Eq. (5.77)
Ther
simplifies to

aoyu" + agX u'y1 = (x u" + u')a,x y1 = 0


T#0, y# 0,
Since
ag * 0, we
get xu" + u = 0. Separating the variables, we get
= -
[ntegrating, we get for x> 0

In |u|= -In x, or
u'
Integrating again, we get u = In x.

Therefore, y2 = uy1 = Y1 ln x. Since y2/y1 = In x, is not a constant, the two solutions y1,
inearly independent. The general solution in this case is yz are

y(x) =
Ay1 + By2= (A + B In x)y1 =
(A + B Inx)x" (5.78)
where m= (a0 - a)/(2a).
Example
Example 5.45 Find the solution of the differential equation 4x*y" + y ==0.
Solution Here, ag = 4, aj = 0, az = 1. The auxiliary equation is

agmm - 1) + ajm + a2 = 4m* - 4m + 1 = 0, or (2m 1 = 0.

The equation has the double root =1/2. The


m
general solution is (from Eq. (5.78))
yx) = (A +B In x) x'2, x>0.

Complex roots
Let the roots of the auxiliary equation (5.74) be a complex conjugate pair, m =
p t iq. Then the
solutions are given by
x = xDi9 = xP x*ig = xP(elnn)tig
=
xPetiqinx =
xP[cos (q In x) t i
(q ln x)], sin x> 0.
Therefore, we can take the two linearly independent solutions as
i) xP cos (q In x), and y20t) = x" sin (q In x).
(5.79)
Example 5.46 Find the general solution of the equation 4xy" + 8xy + 17y =0.
Solution Here, ao =4, aj = 8 and a = 17. The auxiliary equation is

agm(m - 1) + ajm +a2 = 4m + 4m + 17 = 0.

he roots of this equation are m = -4tI6-


8
212 -4 l6i = t2i=p t -

iq.
The
general solution is (from Eq. (5.79))
yx) = Ax" cos (2 In x) + Bx"* sin (2 In x).
5.40 Engineering Mathematics

The method considered here is easily applicable for the homogeneous equations. However, Ior non-

Ogeneous equations, finding a particular solution is difficult.


We now discuss a method which can be applied for the solution of general Euler-Cauchy equation
given by Eq. (5.71). For x >0, we change the independent variadie

X =e', or t= In x, x> 0. (5.80)


The casex<0 can also be considered by writing the transformation as

xl=e, or t= In |x (5.81)
his transformation always reduces the Euler-Cauchy equation into a linear equation with constant
in the
cCiets. The solution of this equation can then be obtained using the
methods discussed
preious sections. Finally, the solution of the given equation, in terms of the original variable x, is
obtained by replacingt by In x.
When x =e', t= In x, we have

(5.82)
d2
dx2
or
(5.83)
In operator notation, set D dldx, D'
=
=
d-ldxr, 0= dldt, 0' =
dldt- etc. Then, Eqs. (5.82), (5.83)
can be written as

xD =
0, xD
02 0= e(0- 1) = -

(5.84)
or xDy = 6y. *Dy = 0(8-1)y.
By induction, we can prove that
(5.85)

D"y (6- 1)... [6- (n -

1)] (5.86)
Substituting in the non-homogeneous second order linear
equation(5.71), we obtain the reduced equation as
a 0(6-1) y + a1 6y+ a2 y ao 6"y + (a1 ag) + =

0 y a2y r(e').
-

This is a linear equation with constant coefficients. The (5.87)


methods described in the
can be applied to find its solution. previous sections
Example 5.47 Find the general solution of the
equation 2xy" + 3xy' -

3y =
x.
Solution Using the transformation x =
e, we get (using Eqs. (5.82) and 5.83))

2 dt 3 -

3y =e, or
2 3y=
This is linear, constant coefficient
a
(5.88)
equation. Substituting,
coresponding homogeneous equation is obtained as
y =
e the
characteristic equation of the
2m2+m-3 =
0, or (m- 1)(2m + 3) = 0,
function is y. )
or m =1,- 3/2.
The complementary = Ae' + Be-/2
Linear Diferential Equations 5.41

Let the particular integral be written as y, = ce*". Substituting in Eq. (5.88), we obtain

(18 +3 3)ce3' = e3', or c = 1/18.

The particular integral is y, = eS"/18.


general solution is y() = Ae' + B e + e
18
Substituting e' =x, we get the general solution as

+ y(x) = Ax + B
18 x
Example 5.48 Find the general solution ofthe equation y" + 5xy' +3y = In x, x> 0.
Solution Using the transformation x =e', we obtain

-45 +3y =in (e'), or 43y=t. (5.89)


The characteristic equation of the corresponding homogeneous equation is
m + 4m +3 = 0, or (m + 1)(m +3) = 0, or m= -1,-3.

The complementary function is y.() = Ael+ Be


Let the particular integral be written as y, = Cit + C2 Substituting in Eq. (5.89), we get

4c1 + 3(°it + C2) = t.

Comparing the coefficients of t and the constant terms on both sides, we obtain 3cj = 1 and
4c+3c2 =0. The solution is cj = 1/3, c2 = --4/9.

The particular integral is yp =

The general solution of the given equation is

yt)=Ae- + Be-3 +:

Substituting e' = x, we get the general solution as

Example 5.49 Find the general solution of the cquation y" -Sxy + 13y =30r
Solution Using the transformation x = e', we obtain

-
dt2
5 + 13y= 30, o dt2
+ 13y =30 (5.90)
The characteristic equation of the corresponding homogeneous equation is

m- 6m 13 0tV36 52 6t4
+ =
0, or m =
= 3 + 2i.
2
The complementary function is y) e" (A 2r +B sin
= cos 2/).
Let the particular integral be written ce".
as
y, =
Substituting in equation (5.90), we obtain
(4 12 + 13)ce = 30e", or c =6.
5.42 Engineering Mathematics

Theparticular integral is y, 6e".=

2
ne general solution is y(t) e3(A cos 21 +
= B sin 21) +

Substituting e' = x, we get


In x)] + 6x.
x'[A cos (2 In x) + B sin (2
y(r) =

Example 5.50 Find the general solution of the equation


0
r'y"+ 5xy"+ 5xy'+ y =x*+ In x,
x>

Solution Using the transformation x = e', we get (in operator notation


o(0-1)0-2) + 50(0- 1) + 50+ 1]y = e" +t

e" +t
or
[0-302+20+502-50+50 + 1] y
=

(5.91)
or
[0+20 +20+ 1] y e" + t =

where = dldt.
The characteristic equation of the corresponding homogeneous equation is

m+2m + 2m +1 0, or (m + 1)(m* +m+ 1) =0.

Its roots are m= -1, - tiy3


The complementary function is

yt) = Ae"+ [B cos (/3r/2) + Csin (V3t/2)]e-


Let the particular integral be written as =cie" + C2'+ C3
yp
Then, = 2c1e+C2. y=4cje,y"=8cj e".
Substituting in Eq. (5.91), we obtain
(8+8+4+ 1)ce+ C2t + 2c2 + Cg =e+t, or 21ce+c2t + 2c2 + C3 = e4 + t,
Comparing both sides, we get c1 = 1/21, c2 = 1, c3 = -2.

The
particular integral is y,= e+1-2.
The general solution is

yt) =Ae+ [B cos (/3t/2) +Csin (V3t/2)]e-2+e+t-2.


Substituting e' = x, we get

yis)= 4+Bcos(3In x/2) Csin( n x/2)] +|Inx -2+

Example 5.51 Find the general solution of the equation

y" 3xy+ 3y =
16r + 9x* In x, x> 0.
Solution Using the transformation x = e', we get (in operator notation)

[O(0-1)0-2)-30+3ly =16e' +9te


Or (0 30-0+3)y = 16e' 9te2 +
(5.92)
Linear Differential Equations 5.43

where 6= dldt. The characteristic equation of the corresponding homogeneous equation is

m-3m- m+3 =0, or (m 1)(m + 1) (m - 3) =0. or m=t1,3.


The complementary function is given by y.(t) = Ae' + Be-t + Ce3'. Note that e', which is one of the
inearly independent solutions, also appears as a term on the right hand side of Eq. (5.92). Hence, by
the method of undetermined parameters, we write the particular solution as

p = (cif + c)ed + czte'

We have y= (c+ 2ct+ 2c2)e" + (1 + )cje'


y= (4c + 4ct + 4c2)e" + (2 + 1)c^e'

=(12c+8ct +8cz)e"+ (3 +)ce'.


Substituting in Eq. (5.92), we obtain
[(12c1 + 8ct + 8c2) - 3(4c1 + 4c1t + 4c) - (c + 201t + 2c2) +3(ct + ca)le
+[(3+Tc3 - 3(2 + 1)e3 - (1 + 1)c3 + 3czt]e' = 16e' + 9te2

or -

(c + 3ct +3c)e2 4ce' 16e' + 9te2"


Comparing both sides, we obtain cj + 3c2 = 0, -3c = 9, - 4cy 16. The solution is c = -3, c2 = 1
and c3-4.
The particular integral is y,() =
(1 -3/)e-4te'.
The general solution is y(t) = Ae' + Ber+ Ce3 + (1 - 3r)e2- 4te'.
Substituting x = e, we obtain the general solution as

y(x)= Ax + + Cx3+ (1 - 3 In x)x2 - 4x Inx.

Exercise 5.6
Find the general solution of the following homogeneous differential equations (Assume x> 0 in Problems I to 20).
1. + xy' - 4y = 0. 2. x+4xy +2y 0.
3. *y+ay-y = 0. 4. 9xy" + 15xy' + y = 0.
6. 2xy" +2xy'+ y = 0.
5. 4xy"+ 16xy +9y =0.
7. y3axy'+ y = 0. 8. xy- xy + 5y =0.
10. 9xy"+ 3xy+10y = 0.
9.y+ 3xy+10y =0.
11. y " +2y" = 0. 12. "+xy' - y = 0.
13. y " + 4x2y"+ 2xy -2y =0. 14. y+9x*y" 18ry' 6y 0.
+ + =

15. y " -2xy + 4y =0. 16. y" +3y" 14ry+ 34y =0.
+

18. y+6x"y" +
17. y+3x'y" = 0. 4x*y" -2ry' -4y =0.
19. 4xy + 24x'y" + 43xy" + 19xy' 4y
- = 0. 20.y 6x'y'" 5x*y" -xy'+ y =0.
+ +

Find the general solution ofthe following differential equations (Assume x>0 in Problems 21 to 40).
21. y -2y 2x+6 22. - 3ry'+3y = 2+3 In X
5.44 Engineering Mathematics

15x.
23. y" + 2xy' 6y
=

x*y" +2xy-2y =6x - 14. 24.


25. 24r In x.
x*y" +2xy' cos (In x). y + Sxy' 5y
- =

= 26.
27. =x.
4xy"+ y= 25 sin (In x). 28. xy"-3xy + 4y
29.
4xy+ 16xy + 9y =19 cos (ln x) + 22 sin (In x).
30.
*y"+2xy -2y (In x). = x sin 31. y-2xy'-4y 6r+ 4 In =

32.
*y 8ry"+ 5xy -5y =42x
t+
33. r'y +6ry" 12y 12/.
- =

34.
'y"- 3xy"+7xy 8y 3r + 8x.
35. 4ry" + 12r-y"+ xy +y = 50 sin (ln x).
36. (3x +
1)y" + (3x +1)y' + y =6x. 37. (x+2)'y" + (x + 2)y" +(x + 2)y' - y = 242
38.
*y 6ry" +2y"-4xy + 4y =10/x.
+ 39. 4xy+ 16x'y" - xy"+ 9xy' -9y = 14 +1,
40. y+ 6xy" + 12xy" + 6xy + 4y
n d the
=2/x*.
solutions of the
following differential equations, which satisfy the gVen
41.
2xy"+ 3xy co
y =x, y(1)
-

1, y(4) 41/16. = =

42. 4xy" + y = In x, x> 0,


y(1) 0, yle) 5. = =
43. y" -3xy' + 3y 5x- x , =
y(1) =1, y(1) =
3/2.
44. *y"- xy' +2y =6, y(1) 1,
45. y'(1) 2. =
=

x*y"+ 3xy' +10y 9x, y(1) 5/2, =


y'(1) =
= 8.

5.5
Operator Methods for Finding Particular
Insection 5.3.1, we have introduced the
Integrals
write differential operator D, where D= dldx. For
example, we can

L(y) = a
cy =
(aD+ bD
d2
dx +
c)y =
F(D)y.
Since D is a differential operator, its inverse D defines
D'Df) =f). the integral operator, such that
In this section, we
develop symbolic short cut
methods for finding a
non-homogeneous equation with constant coefficients. particular integral of a linear
Consider the linear
non-homogeneous equation with constant coefficients
L(y) =ao +a + +
an-1 dy + any =r(x) (5.93)
or
L(y) (aoD" +a,D"+..+an-1D
=

+a,)y =F(D)y =r(x)


where F(D) agD" + a1D"* * ..+dn-1D* an»
= (5.94)
Eq. (5.94), we write the particular integral as and ao» a1, .
.

,
an are constants. From
ypx) = [F(D)I'r ().

In the following, we develop methods for (5.95)


finding lF(LD)r () for
particular cases of r (r).
Linear Diferential Equations 5.45
5.5.1 Case r(r) =ear

When y= e"", we have

F(D)y =(a,D" + a D- +.. +


an-1D +a,)e .

=
(ag" + a1a-+.. .+ a,-a+ a,)ea" =
Case Fl) #0
F(0)e*
We may now
symbolically write this equation as

y =
[F(D)I'F(a)e" F(a)[F(D)]lr =

since F(a) is a constant. We can further write

Fay=F(D)]"e"", or [F(D)]'e" = = e

Hence, if r () = e"", we obtain

plr) [F(D)I" e"


=
=
F F() +0. (5.96)
F
,

We can verify that this result is true.


Operating with F(D) on both sides, we get
F(D)»(x) F(D) F =

FO(D)e*
F a Flae" =e
Example 5.52 Find the
general solution of the differential equation y" -2y' 3y
3e4 -
=

Solution In operator notation, the


given equation is (D 2D 3)y 3ea, The characteristic -
-
=

equation of the corresponding homogeneous equation is


(m-2m 3) (m 3)(m + -
= -

1) =0. Its roots are m = -

1, 3.
The complementary function is
given by y. (x) = A+ Be%.
We have F(D) = D> -2D 3 . The particular integral is

ypx)=[F(D)Ir rr) ={D?-2D-3r (3e)= =-,


The general solution is

yx) = ye()+ yp*) =Ae+ Be-

Example 5.53 Find the general solution of the equation y-2y"- 5y' + 6y =
4 -*
Solution The given equation in operator notation is
F(D)y= (D -2D- 5D + 6)y = 4e"- e, where F(D) = - 2D-5D +6.
he characteristic equation of the corresponding homogeneous equation is
m-2m-Sm +6=0, or (m- 1)(nm +2(m - 3) = 0.
Ihe roots of this 1, 2, 3. The complementary function is
equation are m = -

yx) = Aet + Be-x + Ce3


The particular integral is
5.46 Engineering Mathematics

pt) = [F(D)I(4e- e2)


= [F(D)]"(4e) - [F(D)r'e*

FTT-F
The general solution is

y(x) = y (x) + »(x) = Ae* + Be- + Ce3 + +

Before we discuss the case when F(a) = 0, let us derive the following result

F(D)Ietrea"] = e F(D + a) g). (5.97)


By Leibniz theorem, we have
D'[g)] =(D"¢"*)8 +"C(D-a*)(Dg) + .. . + e*(D"8)
=
a"eg+"Ca"-la*Dg +"Ca-*(D°s)+ ..+ e*(D"8) .

= eD"g + "C,(Dg)a+ "C(Dg)a+... +a"g]


(by reversing the order of terms and using the result "C, = "Cr-+)

= eD" +"C,aDn-l+"C;a*Da2+... + a"lg


=
e"[D + a]"g.
Substituting the expressions for n = 1, 2,... on the lefthand side of Eq. (5.97), we obtain

F(D"g)] =
[a%D"+a,D +a2D+...+ a,(e"g)
aD"("s) +a,D(e"g) + ... + a,(e"g)
=
ay e(D + ayg + ae*(D + a)"g +...+ a, e"8
=
e "a (D +a"+ a(D + a+...+anl8
= eF(D + a)g.
Case F ( ) = 0.

Let us now consider the case F(a)=0. From the theory of polynomial equations, (D- a) is a factor
of F(D). If F'(a) also vanishes, then (D-a)" is a factor of F{D). If F(a) = 0 = F'(a) = .. .
F-D (a), F'(a) # 0, then (D - a is a factor of F(D). Then, we can write
F(D) = (D - ayG(D), G(a) * 0.
(5.98)
Let us now write the particular integral of F(D)y = e as

Yx) =
[F(D)I"e" [D - ayG(D)F
=
=
[(D -

ayr[GD)rlea«
[(D- ayrIG(a)r'ea" (since G(a) * 0 and using Eq. (5.96))
Ga(D- aT'|e",1] = iT"[(D+ a - a1'[i

(using Eq. (5.97)


5.47
Linear Differential Equations

1 ex (5.99)
Gay G(a)
since D represents integration r times.
Therefore, when F(a) = 0, F ( a ) # 0, we have F(D) = (D - a)G(D) and the particular integral or
FD)y = e i s given by

(5.100)
yp(x) G(a)
Generalization to the case r (x) = e*h(x).
Irrespective of whether F(a) vanishes or does not vanish, the above result can be extended to the case
r(x)= "h(x). We have the particular
integral in this case as
px)= [F(D)IlE*h(x)] (5.101) =
¬"*[F(D + a)]h(x)
using Eq. (5.97). Now, [F(D + a)]hx) can be evaluated when h(x) is of some particular forms.
Example 5.54 Find the general solution of the equation y"+ y- 6y = 5e3.

Solution The equation in operator notation is


(D + D -

6)y =
5e*, where
F(D) =
D + D -

6 =
(D + 3)(D -

2).
The characteristic
equation of the corresponding homogeneous equation (D* + D- 6)y = 0 is
m +m 6 0, or (m + 3)(m -2) 0, or m 2, 3.
-
=
= = -

The complementary function is ye (x) Ae*


= + BeS*
Now, F(m) =
m*+ m
-6, F(-3) 0 and F'(-3) -5 # 0. Therefore,
= =

ypx) [(D +3)(D 2)1(5e*) 5(D + 3)-lI(D 2)'e]


= -

= -

=
5(D 3)(-5)e* - (D +.3)'[e
+ = .

1] (using Eq. (5.96)


= -

(D -3+ 3) 1 =- eD'(1) = -
x e*. (using Eq. (5.99))
We might have also used the formula (5.100) directly where G(D) = D -2. The
general solution is
yx) = ye(a) + ypl*) = Ae + B * - x e-3x

Example 5.55 Find the general solution of the equation 4y" 4y'+ y e*/2 -
=

Solution The characteristic equation of the corresponding homogeneous


Solution
equation is
4m- 4m + 1 =0, or (2m 1) = 0. Its roots are m =
1/2, 1/2.
ne
complementary function is ye (x) =
(A + Bx)e*i, We have
F(D) = 4D - 4D + l =(2D -1), and F(1/2) = 0.
The
particular integral is

px) =
(2D -

1)-(e*2. 1)
)-o
8
5.48 Engineering Mathematics

The general solution is


y(u) =
(A + Bx)e* + (r*e*)/8.
x/3
EXample 5.56 Find the general solution ofthe equation 9y" +3y-5y+ y 42e" + =

Solution The characteristic equation of the corresponding homogeneous cquation


9y"3y 5y +y = 0 is

9m +3m -
5m + 1 =
0, or (m + 1)(3m -

1* = 0.
The roots of this equation are m = -

1, 1/3, 1/3. The


complementary function is

Ye(x) Ae""+ (Bx + C)e*3


=

We have F(D) = 9D+3D- 5D + 1 =(D + 1)(3D - 1) and F(1/3) = 0.


The particular integral is
yplx) [(D + 1)(3D 1T(42e* +64e*
= -

=
[(D + 1)(3D -

1)1(42e") + [(D + 1)(3D -

1)1"64e).
Since F(1) # 0 and F(1/3) =
0, we obtain
ypr)= [(1 + 1)03 1 1(42e") (3D - 1)t(D 1)(64e*3+ +

64
e(3D -1)- 4/3)

.)- (1)

-2 *p-2(1)= 2e'+re
The general solution is

x)= Ae+ (Bx +C)e34e+x*ex3


Example 5.57 Find the general solution of the equation 16y" +8y' + y =
48xe4
Solution The characteristic equation of the corresponding
homogeneous equation is
16m+8m + 1= 0, or (4m+ 1) =0. Its roots are m
=- 1/4, -

1/4.
The complementary function is ye (x) (Ax + B)ev/4 =

We have F(D) =16D + 8D + l (4D + I) and


= F(- 1/4) =0.
The particular integral is

ypx) =
(4D +
1)*(48xea) 48e-s/44 =

=
48e-4 (4D)* (x) =
3e4 D-(x) re =

The general solution is y(x) =


(Ax + B)e"/4 + ex/4
Linear Differential Equations 5.49
5.5.2 Case r(")
=
cos ax or
sin ax.
der first, the case
Consider
when F(D)
When flr) = cos ax, we have contains even
powers of D.
Df - a cos ax, D*f =(-
a23 cos ax, D°f= (-a* cos x,
Let FD)y lag(D " +a,(D y=l =

Now, let y = cos x , then ag(D y*2 +... a, ]y. +


+

FD) cos ax =[ag{D°" +


aj({Dy= + .+ a,) cos axx ..

la(-a* +a- a*y*+...+a, ) cos ax =F(- a) cos ax


Case F-a) #0 (5.102)

From Eq.(5.102), we symbolically write


cos
ax =[F(D )I[F-a) cos ax]
or
=
F(-aF(D)1l cos ax

TF(D)}cos ar = COS X
Therefore, the F-a)
particular integral of the equation F(D")y =cos ax is
given by
COS ax
ypx) [F(D})]' cos ax =F-a2F(-a)
=

F(-a) #0. 0. .

(5.103)
It is easy to show that similar formula
holds when r(x) sin That is, if
=
ax. F(D")y = sin dx, then
p(x) = [F(D )] sin ax =- Sin ax
(5.104)
F-a)
When odd powers of D also exist in
F(D), we can follow the same procedure to obtain y,(x).
Let F(D) =
Fi(D<) + F2(D), where F2(D) contains odd powers of D. Then
[F(D)+Fg(D)] cos ax =[F-a") + Fz(D)] cos ax
Since F(D) has constant coefficients, we obtain

IF(D)+ FaD)1 cos ax = [Fi- a) + Fz(D)] cos ax,


(5.105)
We now
simplify Fi(- ?) + FD) and multiply it by F3(D)IF;(D)T', where F(D) contains odd
powers of D, such that [F(-a2) + F(D)]F;(D) contains only even powers of D. Formula (5. 105) is
applied and the procedure is repeated to obtain yplx). We ilustrate this technique through examples.
xample 5.58 Find the general solution of the equation y+4y = 6 cos x.

Solution It is easy to verify that the complementary function is

A 2x + B sin 2x.
ye (x)
= cos

We
have F(D?) = D? + 4 and r (x) = 6 cos x, that is a = 1. Since F(-a) =
-a' + 4 and

F-1)=-1+4=3 #0, we have


cos ) =OCOS= 6 cosr. 2 cos X.
p(x) [(D 4)]- (6
= +
F(-a*) F-1D
The general solution is y(r) = A cos 2r +B sin 2r+2
cos x.
5.50 Engineering Mathematics

16 cos 2x.
Example 5.59 Find the general solution of the equation 2y"+y-y=
1
equatOn
SOlution The characteristic equation of the corresponding homogeneous
0.
2m+m- 1 =0, or (m + 1)(2m - 1) =

Its roots are m =- 1, 1/2. The complementary function is

Be"",
X)= Ae" +
we have F{D) = 2D* + D - 1, r(x) = 16 cos 2x. Therefore, a = 2. Using Eq. (5.l05), we get

Ypx) =
[(2D +D -

1)j'(16 cos 2x)D -1] cos 2x =


16[2(-4) +

cos 2x
16(D 9)] cos 2x 16(D + 9)N(D + 9(D 9)1
-

= -
=

1
=
16(D+91(D? 81)]' cos 2x -

(D+9)
8: cos 2x = -

9cos 2x-2 sin 2x)


The general solution is

y(x) = Ae"A+ Be2- 9 cos 2x - 2 sin 2x).


85
Example 5.60 Find the general solution of the equationi y" - 5y' + 4y = 65 sin 2x.

Solution The characteristic equation of the corresponding homogeneous equation is

m -5m+4 0, or (m - 1)m - 4) = 0. Its roots are m = 1, 4.

The complementary function is y.t) = Ae* +


Be
We have F(D) = D* - SD + 4, r (r) = 65 sin 2x. Therefore a = 2. Using Eq. (5.105) we get th
particular integral as
yx) = (D- 5D + 4)(65 sin 2x) = 65-4-5D + 4](sin 2x)

=-Dsin2x) cos = x.

since integral of sin 2x is (- cos 2x)/2. The general solution is

x ) = Ae* + Be4a + cos 2x

Example 5.61 Find the general solution of the equation y" - y"+ 4y'- 4y = sin 3r.

Solution The characteristic equation of the homogeneous equation is

m3m2+ 4m -4 = 0, or (i - 1)(m" + 4) = 0. Its roots are m = 1, t 2i.

The complementary function is y,(x) = Ae* + B cos 2r+ C sin 2x.

We have F(D) = D' - D' + 4 D - 4 = (D -D(D- + 4), r(x) = sin 3r, a = 3.

Using Eq. (5.105) we get the particular integral as

y,x) = |(D - 1(D + 4)I(sin 3x) = I(D - 1D(-9 + 4)|' sin 3


Lin ear Dif fere ntia l Equ atio ns 5 .51

=- ~ (D + l)(D + 1)- 1 (D - 1)- 1 sin 3x = - ; (D + l )( D 2 1


- 1)- sin 3 x
=- 1 (D
+ 1)(- 9 - 1)- 1 sin 3x = - 1 ( D + 1) si n 3x
5 50 - _!_ (s in 3x + 3 cos )x) .
- 50
The general solu tion is

y(x) = Aex + B cos 2x + C si n 2x + (3 cos


3x + sin 3x)/50 .
Remark 3
Note that the abo ve resu lts hol d goo d whe .
n r(x) is also of the form cos (ax + a)
or s tn ( ax + b) .
case F(- a 2 ) = 0.
When F(- a 2 ) = 0, we wri te cos ax =
Re (e la x ) and sin ax = Im (e 1ax) and
(5.97). We sha ll illu stra te this tech niq app ly the fo rmu la
ue thro ugh the foll owi ng exa mpl es.
Example 5.6 2 Fin d the gen eral solu tion
of the equ atio n y" + y = 6 sin x .
Solution The com plem enta ry fun ctio
n is Ye (x) = A cos x + B sin x.
2
We hav e F(D ) = D + 1, r(x) = 6 sin x.
2
The refo re a= 1 and F(- a 2 ) = F(- 1)
We write the par ticu lar inte gral as
= 0.
yp(x) = (D 2 + 1)-1(6 sin x) = Im (D2 + lf 1(6e 1x)
= 6 Im {eix [(D + ,)2 + 1r1(1)} = 6 Im {e 1x[D 1 + 2iD r 1
(1)}
= 6 Im {e 1xD- 1[(D + 2i)- 1](1) } = 6 Im {e 1xD- 1
(0 + 2tT 1(1) }
= 3 Im {t e 1xx} = 3x Im {- i(co s x + i sin x)} = - 3x cos x.
The gen eral solu tion is y(x ) =A cos x + B sin x - 3x cos x.
Exa mp le 5.6 3
Fin d the gen eral solu tion of the equ atio
n y" - 4y' + l 3y = l 8e2 r sin 3x.
Solution The cha ract eris tic equ atio n
of the hom oge neo us equ atio n is

m2 - 4m + 13 = 0. Its roots are m = 4 + ✓ 16 - 52


-
2 = 2 ± 3i .
The com plem enta ry fun ctio n is Ye (x) =
e2x (A cos 3x + B sin 3x).
We have F(D ) = D 2 - 4D + 13 and r
(x) = l 8e2x sin 3x.
We write the par ticu lar inte gral as

yµ(x) = 18[ D2 - 13r (e2x sin 3x)


4D + 1

= l8e2x[(D + 2) 4(D + 2) + 13r (sin 3x)


2
- 1

(us ing Eq. (5 .9 7))


= l8e2x[D2 + 9r'( sin 3x) = 18e 2 rllm (D2 + 9r ' (e 3 '-')}
= l8e2x (Im e 3'x[(D + 3;)2 + 9r ( 1)}
1

= 18e2x {lm e 3'XfD2 + 6iD r 1(l ) }


5.52 E . .
ngmee ring Mathe matics

= 1Be2 f {Im e 3 ixo- 1(D + 6if 1(1)}


1
= 1Be2x {Im e 31x o-1(0 + 6i) - I (I)}= 1Be2x Im 6i Jix }
{ xe

= 3x e 2 r Im {- i(cos 3x + i sin 3x)} = - 3xe2x cos 3x.


The genera l solution is

y(x) = elx(A cos 3x + B sin 3x) - 3x e2x cos 3x.

5.5.3 Case r(x) = xa, a> O and integer.


The partic ular integral of F(D)y = xa, is

yp(x) = [F(D) r 1xa


Symbo lically , we expan d the operator [F(D) r 1 as an infinite series in ascend
ing power s of D ao<l
operat e on x 0 .

Exam ple 5.64 Find the general solution of the equation y" + 16y = 64.x2.
Soluti on The complementary function is yc(x) = A cos 4x + B sin 4x.
The partic ular integral is

= 4[1- f; fs: -...]x


+ 2
= 4[ x2 _ ~]

The genera l solutio n is y(x) =A cos 4x + B sin 4x + 4x2 - (1/2).


Exam ple 5.65 Find the general solution of the equation y" + 4y' + 3y = x sin
2.x.
Solut ion The characteristic equation of the corresponding homogeneous equati
on is
2
m + 4m + 3 = 0, or (m + l)(m + 3) = 0. Its roots are m = - 1, - 3.
The compl ement ary function is Yc(x) = Ae - x + Be- 3x_
The partic ular integr al is
yp(x) = [D 2 + 4D + 3r (Im xe
1 2
ix) = Im {e 21x[D + 2i)2 + 4(D + 2i) + 3r (x)}
1

= Im {e 2 ix[D2 + 4(1 + i)D + (Bi - l)r 1(x)}

2
e2ix [ 4( 1 + i)D D ] - \ )}
= Im { Bi - 1 1 + Bi - 1 + 8i - 1 x

_ Im {
-
e 2i x
Bi - 1
[l _ 4(1 _+ i~D + . ..
8, -
J(x)}
(8i + 1)
= Im { (- 6 S)
e2 i x [x- 4(8i + l)(l + i) ]}
(- 65)
Lin ear D(fferential Equ ations 5.5 3

= Im {- J5 (Si+ l)(cos 2x + i sin 2x)[x + ~ (9i - 7)]}


= Im {-J 5 [(c os2 x - 8 sin 2x )+i (si n2 x+ 8co s2x )][ (x- ~n !~ + i]}

= - i
4 25 [ 65x (8 cos 2x + sin 2x) - 28(8 cos 2x + sin 2x) + 36(cos 2x - 8 sin 2x)
l
= - i
4 25 (65 x(8 cos 2x + sin 2x) - 188 cos 2x - 316 sin 2x] .
The general solution is

y(x ) = Ae- x + Be- 3x -


~ (65x(8 cos 2x + sin 2x) - 188 cos 2x
42 5 - 316 sin 2x] .
Example 5.66 Fin d the general solution
of the equation + 3y" = 108x2• iv
Solution The characteristic equation of
the homogeneous equation is
m4 + 3m2 = 0, or m2 (m2 + 3) = 0. Its roots are m = 0,
0, ± ,J3i.
The com ple me nta ry function is Ye (x)
=A + Bx+ (C cos .fS x + D sin .fS x).
We have F(D ) = D4 + 3D2 = D2(D2 +
3). The particular integral is given by

Yp (x) = 108 [D 2 (D 2 + 3)r 1 (x 2 ) = 108


[D- 2 ] ½[l + ~2]-1 (x
2
)

2
= 36 W 2 i[1 - ~ + ~• - . . } 2
) = 360 - 2 [x 2
-;]

X4
= 36 [ 12 - 3X 2 ] =
3 4 12 2
X - X •

The general solution is y(x) =A + Bx+


(C cos .fSx + D sin .fSx ) + 3x4 - 12x2
.

Exercise 5.7
.d h
Fm t e genera1 so1u t'on
1
of the following differential equations.
1. (D2 +S D+ 4)y = l8e2x. 2 (D2 - l)y = 8e3x
· ·
3. (D2 - 3D - 4)y = ex + 6e 5 x. 4. (D2 + D + 2)y = ex12•
5. (D 2 + 3D + 3)y = ?ex. 6. (D2 - 2D + 1)y = Se.i.t + 4e 2.,·.

7. (9D2 -6D + l)y =4e-x. 8. (D 2 - 6D + 9)y = 14e1t .

9. (D 2 + D - 6)y = e 2x · 10. (2D2 - 3D - 2)y = xe-x12 •


11. (D2 -l) y= 6x ex. 12. (4D2 ❖ 9D + 2)y = .,I 21·.
?
13. (9D - + 6D + 1) Y = e-x /3 · 14. (2D2 + 7D - 4)y = xe- 4 ·1·•
15. (D3 + 2D2 - SD - 6)y == 4 ex. 16. (2D3 + 3D2 - 3D - 2)y = \O e! t_

17. (D 3 - 2D 2 - D + 2)y ==
3 18. (JJ 3 -6/J 2 + 12D - 8),Y= 18P 2 t.
£' ·' .
5.54 Engin eerin g Math emati cs

3 + 3D2 - 4D - 12)y == l2xe- 1x.


19. (2D3 - 3D2 + l)y = 16ex. 20. (D
21. (D 2 + 16)y = cos 2.x. 22. (2D2 - 5D + 3)y == sin x.
23. (3D 2 - 7D + 2)y = sin x + cos x. 24. (2D2 - 7D + 3)y == sin 2x.
25. (D 2 + D + I )y = 16 cos x. 26. (8D2 - 12D + 5)y == 16 sin x.
27. (D 2 + 9)y = sin 3x. 28, (D2 + 3)y = cos ,./3x.
2 2x ·
29. (D + 2D + 5)y = e-x cos lx. 30 _ (D2 _ 40 + 5)y = 24e sm x.
31. (D 2 - 6D + 13)y = 28e1 t sin lx. 32. (D - 2D + JO)Y = l6ex cos 3x + 24ex sin 3x.
2

33. (D 3 - 3D2 + D - 3)y = 6 cos x. 34 _ (D3 _ 0 2 + 9D _ 9)y == 30 cos 3x.


35. (D 3 - 4D 2 + 9D - lO)y = 24ex sin 2.x. 36. (4D3 - 12D2 + 13D - IO)y == l6ex12 cos x.
37. (D 4 + 5D 2 + 4)y = 16 sin x + 64 cos 2.x.
38. (D 2 + 25)y = 9x3 + 4x2. 39. (D 2 + 6D + 9)y = 4x2 - 1.
40. (D2 - 2D - 3)y = 2x2 + 6x. 41. (D2 - 5D + 6)y == X cos 2x.
2
42. (D + D - 2)y = x 2 sin x. 43. (D 2 - D - 6)y = xe- 2x.
44. (D 2 + 7D + 12)y = ex sin 2.x. 45. (D 2 + 4D + 3)y = e1x cos x.
46. (D 2 + 3D + 4)y = ex cos ( ..ffx/2). 47. (D 2 + 3D + 2)y = xex sin x.
2
48. (D + 9)y = xelx cos x. 49. (4D2 + 8D + 3)y == xe-x12 cos x.
50. (D 4 + 3D2 + 2)y = l 6x 2 cos x.
51. lf (2D - l)y =e3x, then prove that (D - 3)(2D - l)y =O. Find
the general solution of the secon d equation
and substituting in the first equation obtain the general solution.of
the first order equation.
52. If F(D)y =(D - m)y =r (x), then show that the particular
integral can be written as

Yp(x) = emx I e-mx r(x)dx .

53. Show that y = ! lx r(t) sin n(x - t)dt is the solution of the equation y" + n 2y = r(x).
54. lf u is a function of x, then show that

F(D)x u = x F(D)u + F'(D) u


where F(D) = a 0 Dn + a1Dn-l + ... + an, and a are constants.
1
55. Let a given differential equation be of the form F(D)y =r(x)
=x u (.t ).
Then, using the result in problem 54 prove that the particular integr
al y(t) can be . .
· wntte n as
y(x) = [F(D )r xu(x) =x[F(D )r 14(X)- [F'(D ){F(D )t:?]u (x).
1 1

56. The particular integral of the equation F(D)y = e'nx is


X emx
YP (x) = TI G(m) ' where F(D) =(D - m)G(D ), G(m) ~ O,

x 2 em x
y1, (x) = 2! G(m) 'where F(D) = (D - m) 2G(D ). G(m) ~
0 . etr.
Show that these particular integrals can be wriuen us
Linear D~fferential Equations 5.55

[F (D)J-1e'"" = x [ 1
F'(m)
]e'"" ' F(m) = 0, F'(m) °I' 0

1
[F(D)J- e'"" = x 2 [ F"~m)] e'"", F(m) = O, F'(m) = 0, F"(m) °I' 0, etc.
Use these formulas to evaluate the particular integral in Problems 8, 9, 13 and 19 of this exercise.
57. If F(D) _can be_facto~sed into n distinct factors F(D) = (D _ m,)(D _ m ) ... (D - mn), then show that
2
the parucular mtegral of F(D)y = r(x), can be written as

Y, (x) = A,,••• J,-• •• r(x)dt + A2 e"" J,-•>' r(x) dt + ... + A,e ••• J,-••• r(x)dt

Use this fonnula to evaluate the particular integral in problem 40 of this exercise.
58. The forced oscillations of a mechanical system with periodic input are governed by the non-homogeneous
equation

my+ cy + ky = F0 cos wt,


where m > 0, c > 0 and k > 0. Obtain its general solution when (i) c * O (forced damped oscillations).
(ii) c = 0 (forced undamped oscillations).

5.6 Simultaneous Linear Equations


In the previous sections, we have discussed the solution of a single linear differential equation, in
which y is the dependent variable and x is the independent variable. In this section, we consider the
solution of a system of two linear first order equations in two dependent variables y and y and one
1 2
independent variable t. We shall restrict ourselves to the solution of constant coefficient equations.
For example, the equations

dy2
- - - 5y1
dt
+ 3y2 = e '

dy 1 dy2
(11'') 3 dt
dyl + 2
Yi + Y2 = e-t, - +- - 2y1 + 3y2 = t
dt dt

are two systems of linear, constant coefficient first order equations. These two systems can respectively
be written in operator form as

(i) (6D + 3)y 1 + (5D + l)Y2 = 0, and (ii) (3D + 2)Y1 + Y2 = e-',

- 5y 1 + (D + 3)y2 = e t, (D - 2)y1 + (D + 3)y2 = t

where D = dldt.
The solution of such systems can be obtained by elimi~ating one o~ the variables and solving the
resulting linear, second order equation for the ~econd van~ble. Some~1mes, elimination of one of the
variables may also produce a first order equation for the second variable.
We illustrate the method of obtaining the solution through the following examplt!s .
5.56 Engineering Mathematics

Example 5.67 Find the solution of the system of equations


(5 .106)
dyi + 2 d y2 - 2y, - Y2 = e2,
dt dt

- 0· (5 .107)
-dy 2 +yi - 2y 2-
dt
Solution
Method 1
1
Eliminate one of the dependent variables directly. Differentiating Eq. (5.107) wi th respect to • we get

Y2 + Y1 - 2y2 = 0
where dash denotes differentiation with respect tot. Substituting for Y1 from Eq. (5 ,l06 ), we obtain
2,
Y2 - 2y2 + 2y 1 + y 2 + e 2 ' - 2y 2= 0, or Y2 - 4y2 + 2y, + Y2 = -e
Substituting for y 1 from Eq. (5.107), we get
21 (5 .108)
Y2" - 4 Y2' + 4 Y2 - 2 Y2' + Y2 = -e 21 , or Y2" - 6 Y2' + 5 Y2 = - e
which is a second order equation in the variable y 2 •
The complementary function is (y 2)c = Ae' +Be st . The particular integral is

(y2)p = (D2 - 6D + 5t (-e


1 2
') = 1
e
2
'.

The general solution is y 2(t) = Ae 1 + Bes' + ½e 21


From Eq. (5.107), we obtain

y 1 = 2yz - Y2 = 2 ( Ae' +Be s'+ ~ e 21 ) - ( Ae' + 5Be 5' + je 21


)

3Be s'.
= Ae' -
This procedure can be very cumbersome in general.

Method 2
We eliminate one of the dependent variables after writing the equations in operator notation. We have
(D - 2)Y1 + (2D - l)y2 = e 21 (5.109)
Yi+ (D - 2)y2 = 0. (5 .110)
Operating with (D - 2) on equation (5.110), we get (D - 2)y 1 + (D - 2)2y2 = o.

Subtracting Eq. (5.109) from this equation, we get


[(D - 2)2- (2D - l)]Y2 =- e 2 ', or (D 2 - 6D + 5)y = _ e2,
which is the same as Eq. (5 .108). The remaining solution procedure is same as in method 1.

Method 3
I n this method, we •find the .equations
·
governing Yt and Y2 using the determinant b ,
,l b · . • .
'd .
s, ) cons, enng
(symbolically) the given ~quat1ons as age ra1c equations. Solving the equatio ns (S .l09 ) and (S. ll0)
by Cramer's rule we obtain
Linear Differential Equati ons 5.57
D-2 2D - 1 e2,
2D- 1 D-2 e2I
1 D-2 Yi= 0 D-2
and 2D- 1 D-2
1 D-2 Yi= l 0

or [(D - 2 )2 - (2 D - l)]Y1 =(D - 2)e2', or (D2 - 6D + S)y, =2e21 - 2e21 =0 . (5.111)


and (D2 - 6D + 5)y2 = _ e2t. (5 . 112)
(Note the order of evalua tion of the determ inants on the right hand side. Otherw ise,
the method does
not make any sense). We would have obtaine d the first eqµatio n if we had elimina ted
Y2 in method 1
or method 2. Care mu st be taken to properl y choose the arbitrar y constan ts. If we solve
Eqs. (5.111 )
and (5.112) , we obtain

Y1(t) =Ae' + Be st and y 2(t) =C*e' + D*e st + 1 e 2 '.


These solutio ns should satisfy the given equatio n. Substit uting in either of the equatio
ns, say (5 . 110),
we get

(Ae' + Be st ) + ( C*e' + 5D*e st + ; e 21 - 2C*e 1 - 2D*e 5t - ; e 21 ) =0


or (A - C*)e 1+ (B + 3D*)e5t = 0.
Since this equatio n is to be identic ally satisfied, we get
A - C* = 0, and B + 3D* = 0, or A= C* and B =- 3D*.
We obtain the genera l solutio n as

Y1 (t) = C*e' - 3D*es, 'yz{t) = C*e' + D*es, + j e21


which is same as the solutio n obtaine d earlier.
Example 5.68 Solve the system of equatio ns
(2D - 4)y 1 + (3D + S)y2 = 3t + 2
(D - 2)y 1 + (D + l)Y2 = t.

Solutl·on Multip
• ly t h e secon
· d eq uation by 2 and subtract from the first equatio n. We obtai n

(D + 3)Y2 = t + 2

which is a linear first order equatio n in Y2·


The integra ting factor is e 31 • The solution is

e 31 Yi= f (t + 2)e 3t dt + A =
1
3 (t + 2 ) e 31 - 9e '
3
+A

or _ Ae_3, +
Y2-
1.9 (3t + 5)·
Substituting in the second equatio n, we get
5.58 Engineering Mathematics

(D - 2)y 1 +
1 -3t l (3t + 5)] == t
[-3Ae- 3t + 3 + Ae +9

=2 Ae -Jt + .!.9 (6t -


1 8)
(D - 2)y 1 =2Ae- 3' + t - (3t + 8)
9
which is a linear first order equation in Y1.
The integrating factor of this equation is e- 21 • The solution is

e- 21 y 1 = J[2Ae - 51 + i (61 - 8)e- 2 ' ]di+ B

2 1[ e-21 1 (6e_2 ,)] + B


= - 5Ae_5, + 9 (61- 8) (-2) + 2 (-2)

= - 2 Ae- 51 - ..!.. (61 - 5)e- 2' + B


5 18
or y 1 = Be 21 -
2 Ae- 3'
- _!_ (6t - 5).
5 18
Example 5.69 Find the solution of the system of equations

(3D + l)y 1 + 3Dy2 = 3t + 1


(D - 3)y 1 + Dy2 = 2t.
Solution Multiply the second equation by 3 and substract from the first
equation. We obtain
10y1 = (3t + 1) - 6t = 1 - 3t, or y = (1 - 3t)/10 .
1
Substituting in the second equation, we get
l 1 1
[- 3 - 3 + 9/] + Dy 2 = 2t, or Dy = 2t- lO (9t - 6) = lO (llt + 6)
10 2

which is a linear first order equation in y .


2

Integrating, we obtain 1i 2 6
Y2 =
20 t + lO t + A.
Note that the system has only one arbitrary constant as the ~liminant is a first
order equation. This can
also be verified by writing the determinant of the coefficient matrix, which
is
3D + 1 3D
= lOD
D-3 D
which is of first order only.

5.6.1 Solution of First Order Systems by Matrix Method


The method presented in this section uses some concepts of matrix theory
· We discu th . .
of this method for sol vrng a 2 x 2 system wh'1c h can be generalized
. ·
to an 11 x ss e app11cat1on
n system.
Homogeneous systems
Consider a linear homogeneous constant coeffi cient 2 x 2 system of the
form
Linear D~fferential Equations 5.59
I
Yl :: a II YI + a 12 Y2 (5.113)
I
Y2 == a21 Yi + a22 Y2 (5.114)
where Y1, Y2 are the dependent variables, t is the independent variable and a 11 , a , a , a are
12 21 22
constants. Denote

y == [Yt ]• A == [all a,2 ]·


Y2 a21 a22

In matrix notation, we can write Eqs. (5.113), (5.114) as


y' == Ay. (5.115)
Note that a higher order, constant coefficient homogeneous equation can be reduced to this form. For
example, consider the second order equation y" + ay' + by == 0. Denote y == y 1 and write
Y1 = Y2, <Y2 = y')

Y2 =- ay' -by= - ay2 - by1, (Y2 =y")


or (5.116)

Therefore, the equation y" + ay' +by= 0 is equivalent to the first order system given by Eq. (5.116).
We know that the scalar equation y' =my has the solution y =ce mt. Therefore, for the solution of
Eq. (5.115), we examine a solution of the form
y = e;.'x (5.117)
where x = [x1 , x 2 f, or equivalently y 1 =e;.'x 1, and y 2 = e;.1x 2•
Substituting Eq. (5.117) into Eq. (5.115), we obtain
Ae;.'x = Ae;.,x = e~'Ax.
Cancelling eM, we get
Ax=AX (5.118)
which is an algebraic eigenvalue problem. The eigenvalues are the roots of the characteristic equation
a 11 -A a12
I A - Al I = 0, or
a22-A
=0

or

Note that the coefficient of A is equal to - (trace of A) and the constant term is I A I- The roots of
this equation, that is A = Ai, Ai are called the eigenvalues of A. The eigenvalues may be real and
distinct, real and equal or a complex conjugate pair. We assume that the system has the complete set
of eigenvectors, that is, in the present case the system has two linearly independent eigenvectors. Let
the eigenvectors be denoted by xO> and x< 2>. (If 111 :t- Ai, then linear independence of x(I >, x<1> is
guaranteed). Therefore, we obtain the two linearly independent solutions as
(5.1 19)
S.60 En1<im~erinx Mmhemaric:.,

The general solution is


(5.12 0)
• • A ).11x (I>+ B,e ).21 x (2) •
y(t) = A,y, + 81Y2 = ,e . h
Let x<'>= Ix,,. X12f ond x<2> = lx21, X22f, Componcntw1.sc, n write t e so lution y(t) as
we ca
). I )..71 (5.121)
y 1 (t) = A1c x 11 + B1e ).21 X2 1, Y2 (t) -- A 1e , x,z + B,e
).11
X22 ·
.
The method can be extended to an 11 x II system of linear fficient) first orde r equations .
(conS tant coe
E xornplc 5.70 Fmd . .
the general solution of the homogeneou s linear system
Yt = - 2y, + Yz, Y2 = Yt - 2Yz.
Solu tion In matrix notation, the given system can be written as

y'=
[-21 -2l] y=A y.
where Y = [yi, y2] r. Substituting y = e ). I x and cancellin
. ). ,
g e , we Obtam
· the eige nval ue prob lem
Ax = Ax, that is

(
-2 l][x1] = A[X1 ]· (5.12 2)
1 -2 x2 Xz
The characteristic equation of A is given by
-2- A 1
=O, or (2 + A) 2 -1 =0, or A2 + 4A + 3 = 0.
1 -2- A
The roots of this equation are A= - 1, - 3.
For A = - 1, we get from Eq. (5.122)

- x 1 + x2 = 0 and x 1 - x2 = 0.
The solution is x1 = x2 , so that we can take x< 1> = [1 lf
For A= - 3, we get from Eq. (5.122), x + x = 0, so that
1 2 we can take x<2} = [1 - lf.
Thes e two vectors, x0 >, x< 2> are linearly independent. The
general solution of the syste m is
y = A1e- 1x<l) + B1e- 31 x<2>

or

Componentwise we can write the solution as

Y1 = A 1e -I + 8 1e-31, Yz = A1e- 1 - B1e-31•


Example 5.71 Find the general solution of the linear homogeneous syste
m
I I
Y1 = - ay1 + ayz, Yz ;:; - ay1 - ayz, a~ 0.
Solu tion In matrix notation, the given system can be written as

-a
y'=
[- a
Linear Differential Equations 5.61
Substituting Y = eA.'x a nd cancelling eA.', we obtain the eigenvalue problem Ax= AX, that is

[=aa a
-a
][Xt] = A.[x, ]·
X2 X2
(5.123)
The characteristi c equation of A is
-a -A a
-a _ a _ ).. = 0, or (a + A. ) 2 + a 2 = 0 and its roots are).. = - a ± ia = -a(l + i).
For).= - a(l + i), we get from Eq. (5.123)

ai.x1 + ax2 = 0, - ax 1 + aix2 = 0.


The solution is x2 = - i.x 1. We can take x<l) as x<l) = [l - i ]7.
For A= - a(l - i), we get from Eq. (5.123)

- ai.x1 + ax2 = 0, - ax 1 - ai.x2 = 0.


The solution is x2 = i.x1. We can take x<2> as x<2> = [ 1 i ]7.
Therefore, the general solution is
y = A1e-a(l+i)1x( l> + B1e-a(l-i)tx( 2)

= A,,-a, (cos at - sin al)[_'.]+ B,,-m (cos at+ sin al)[:]


i i

cos at - i sin at] + at + i sin at ]


= A 1e-a, [ . . B 1 e-at [ cos
. .
- sm at - i cos at - sin at + , cos at

= (A1 + B1 )e-at [ cos


. at] +
-sin at
(B1 - A1)ie-at [ sin at]
cos at
= C*e-at [ cos
. at] + D•e - at [sin
-sm at cos at
at]
where C* = A1 + B 1, and D* = (B 1 -A 1)i. Componentwise, the solution is
y1 = e-a'(C* cos at+ D* sin at), y 2 = e-a'(- C* sin at+ D* cos at).

Non-homogeneous systems
Consider now a non-homoge neous, linear constant coefficient system of equations of the form

(S. 12 ~)

(5 .125)
or y' = Ay +- h (5. 126)
where A, y are as defined earlier and h = [/i 1(t) lri(t)f. The solution of the system is y = Ye + Yp where
Yc is the complement ary function and yP is the particular integrnl. The complemen tary functio n y, is
the solution of the homogeneou s equation which can be obtained by the methods described above.
The particular integral Yp can be obtained by the method of undetermine d coefficients, if the co mpork!nts
of h(t) are simple functions like a polynomial, exponential, sine or cosine function. Ho wever. in
general, we can use the diago11alisatio11 method to find yp · We now illustrate both these me thods.
5.62
Engi neer ing Math ema tics

562 M
· · ethod of Undetermined Coefficients to Find the p ar ticular Integral
Sinc e the meth od is strni ghtfo rwnr d we illustrate
it throu gh exam ples .
E I .
xam p es 5.72 Fmd .
the gene ral solution of the linea f uatio ns y' = Ay + h, whe re
r system O eq

2]• h = [Gt + 12t +


3
2

4t 2 +3t +6
9]· and y = [~~ l
Solu tion Con side r the corr espo ndin g hom ogeneous equa tion
canc ellin g c ).', we obta in the eige nvalue prob lem
y' =Ay. Subst1. tutin. g Y =e Al X an d
Ax = AX, that is

[l 2][x']
4 3 X2
= ).[x' ]·
X2
The char acte risti c equa tion of A is

1 - }. 2
= 0 or ). 2 - 4A- 5 = 0, and its root s are A= -1, 5.
4 3-} . '

For A = - 1, (A - )J)x = 0 give s the equa tions


2x 1 + 2x2 = 0, and 4x 1 + 4x2 = 0,
who se solu tion is x 1 = - x . The eige nvec tor can
2 be take n as x< 1> = [l - lf.
F or A= 5, (A - ll)x = 0 give s the equa tions

- 4x 1 + 2x2 = 0, and 4x1 - 2x2 =0


who se solu tion is x 2 = 2x1. The eige nvec tor can be taken as x<2> = [l 2f.
The com plem enta ry func tion is give n by

Ye = c 1e- 1x< 1> + c2e 51 x<2>.


Sinc e the elem ents of h are poly nom ials, we write
the parti cula r integ ral as
Yµ = d t 2 +et + f.
Diff eren tiati ng, we have y; = 2dt + e.
Sub stitu ting in the diffe renti al equa tion, we obta
in
2d t + e = A(d t 2 + et + f) + h.
Com pari ng the coef ficie nts of t 2, rand the cons
tant term, we obta in

Con side r now the first syst em. We have the equu
tions
Line ar Diffe renti al Equa tions 5.63
d 1 + 2d1 = - 6 4d + 3d
or . ' 1
The second syst em give s the equa tions2 = - 4, whose solution is d 1 = 2, and d1 = - 4·

or + 2 e1 = - 8 , 4 e1 + 3e2 =- 11, whose solu tion is e


e1
1
= 2/5, and e1 = - 21 /5.
The third syst em give s the equa tions

or !1 + 2h = - 4 315, 4/1 + 3fi = - 51/5, whose solu


tion is/1 = 27/2 5, and fi = - 121/25.
The general solu tion of the give n syste m is

Y =Ye + Yp = c1e- 1x 0 > + c e 51 x< 2> + dt 1 +et +


2 f

= ce-'[-11] + c es,[21] +
1 2 12 [ 2] + .!_ [ 2] + _1 [ 27]·
- 4 5 - 21 25 - 121
Com pone ntwi se, the solu tion is

YI = C1e- l + 5
C2e ' + 2t 2 + (2t/5 ) + (27/25),
Y2 =- c1e_, + 2c2 e 5 ' - 4t 2 - (21t /5) - (121/25).
Example 5.73 Find the gene ral solu tion of the linear syste m of
equa tions

Solution The eige nval ue prob lem corresponding


to the hom ogen eous equa tion is Ax = AX, that is

-4
-7] X = AX.
The char acte risti c equa tion is give n by

5-A -7
= 0, or ,12 - A - 6 = 0, and its roots are A= 3, - 2.
2 -4- A
For A= 3, (A - Al)x = O give s the equations

2.x 1 - 7x2 = 0, and 2x 1 - 7x2 = 0


whose solu tion is x = 2.x /7. The eige nvec tor can
1 1 be taken as x<n = [7 2)7.
For A=_ 2, (A _ Al)x = O give s the equations

7x 1 - 7x2 = 0, 2.\· 1 - 2x2 =0


5.64 Engineering Mathematics
. . . 2 l) r.
who se solutJon 1s x 1 = x 2• The eigen vector can be ta ken as x( > = [1
The complementary function is given by

Yc= c,e 31 x ( I) +c2 e -


21 x< 2>.

As m the scala r case, we write the particular integral 1 W have y' = de' . Subs titut ing in
as Yp = de · e ' P
the given equation, we have

d e1 = A de 1 - [ !} 1

Canc ellin g e', we obtain

or 4d1 - 7d2 = 2, and 2d1 - 5d2 = 4.


The solution of these equations is d = - 3 and d =
1 2 - 2, or d = [- 3 - 2f.
The general solution of the system is

y =Ye + Yp = c1e3'x(l> + c2e-2'x<2> + de'

Example 5.74 Find the general solution of the linea


r system of equations
-1
y'= Ay + b =
[ 2
Solution The eigenvalue problem corresponding to
the homogen eous equa tion is Ax = Ax, that is

[-~ _!]x=h
The characteristic equation is

-1- A 3
2 -2- A
=0, or ).2 + 3A - 4 = 0, or A = 1, - 4.

For A = 1, (A - Al)x = 0 gives the equations


- 2x 1 + 3x2 = 0, and 2x 1 - 3x = O
2
who se solution is 2x1 = 3x2. The eigenvector can be
taken as xO> = [3 2]7.
For A= - 4, (A - Al)x = 0 gives the equations

3x1 + 3x2 =0, 2x1 + 2x2 =0


who se solu tion is x 1 = - x2• The eigenvector can be
taken as x<2> = [ 1 _ 1)7.
The com plem enta ry function is given by
Linear D(fferential Equations 5.65

Yr= c,e'x (t) + c2 e- 4 'x< 2l.


We note that e- 4 , occurs both in yr and on the right hand side. Hence, as in the scalar case, we write
the particular integral as
Yp = (dt + e)e- 41

We have
substituting in the given equation, we obtain

(-4dt + d - 4e),-4 ' ; A(dt + l!)e- 4 ' + [ }-•,. ~


Comparing the coefficients of te- 4 ' , we get
-4d = Ad.
Hence, dis the eigenvector correspondi ng tu the eigenvalue ).= - 4. Hence, d = px<2>, where pis a
constant. Comparing the coefficients of e-41 , we obtain

d - 4e; Ae + [ ~]

or

or 3e 1 + 3e2 =p - 4, 2e 1 + 2e2 =- (p + 1).


Therefore, 2p - 8 = - 3(p + 1), or p = 1. Hence, d = x<2>. With this value, we get e 1 + e2 = - 1. We
can choose e 1 = - 1, e2 = 0, so that e = [- 1 Of. The general solution of the given system is
Y =Ye+ Yp = c1e 1 x0 > + c2e- 41 x<2> + (dt + e)e- 41
= c 1e' x<l) + (c 2 x<2> + e)e- 4 ' + dte- 41

5.6.3 Method of Diagonalisa tion to Find the Particular Integral


Let the matrix A of the non-homoge neous system y' = Ay + h, have a complete system of eigenvector s.
That is, if A is an n x n matrix, then there exist 11 linearly independent eigenvector s x<1>, xr2>, ... x<n>_
If the eigenvalues ).1 of A are distinct, then linear independenc e of the eigenvector s is guaranteed. Let
x denote the matrix of eigenvector s

(5. 127)
Since xU>, i = 1, 2, . . . , 11 are linearly independen t, x 1s no n-s ingular and x- 1 exists .
Premultiplying y' = Ay + h by x- •, we get
x- 1 y' = x- 1Ay + x- 1 h. (5. 128)
Let Y = xu. We have, y' = xu'. Substituting in Eq. (5.128), we get
5.66 Engineering Mathematics
(5.129)
1
x- 1xu' = x- 1Axu + x- h.
. ) d' onalises the matrix A, that is
We know from matrix theory that the matrix x (of eagenvec~ors iag . envalues of A. Therefore,
th
x-l Ax = D, where D is a diagonal matrix with its diagonal entnes as e eig
we obtain from Eq. (5.129)
(5.130)
g=X - lh .
u' = Du+ g, where
Now, Eq. (5.130) is of the form

0 0 0 U1 81
A.1
"' '
'' 2 =
0 A.2 0 0 U2
+
82 (5.131)

u'n 0 0 0 An Un 8n

Therefore, Eq. (5.130) degenerates into n independent scalar equations


(5.132)
u; = A. 1 u1 + g 1, j = 1, 2, ... , n.
The method of finding the solution of these first order equations was discussed in the previous
chapter. The solution of these equations can be written as

u1 = c1e''' + ,•,•[J g1e_,,, dt]. (5.133)

The solution of the given non-homogeneous system is then obtained from the equation Y = xu.
Example 5.75 Find the general solution of the linear system of equations

y'=Ay+h=[:
(see Example 5.73)
Solution From Example 5.73, we have

x = [x 0 > x< 2>] = G :J ~


and x- =
-1]
1
[-~ 7·

Now,
-1][2] ,_ 1[-2]
7 4
e - - -
5 24
et
.

The eigenvalues of A are A. 1 = 3 and Ai = - 2. Eqs. (5.132) become

u1=A.1u1+g 1 =3u 1 + ~ e'


24 et.
U2 = A.2u2 + 82 = -2u2 - 5

The solutions of these equations are u, = c,eJ' - 51 e'' "2 = c-,e-2'


- - .§.
5 e' • respect·1vel y. Hence
Linear Differential Equations 5.67

y =XU= G
which is same as the solution obtained in Example 5.73.

Exercise 5.8
Find the solution of the following systems of equations using the elimination method.
1. y; =2y1 + Y2, Y2 =Y1 + 2y2, 2. Yt =Y1 + Y2, Y2 =9y, + Y2•
3. y; = Y2, Y2 = - 9y1, 4. YI = 2y, + Y2, Y2 = - l8y1 - 7y2,
s. y; + Y2 = 4 sin t, Y2 + YI = 8 cost. 6. y 1 + y 1 + 3y2 =4e- 1, y 2 + 4y 1 - 3y2 =St.
7. Yt + 3y1 + Y2 = 6e', Y2 - 5y1 - 3y2 = 3e-'.
8. y 1+ 4y 1 - Sy2 = 16 sin t, y 2 + 5y 1 - 4y2 = e'.
9. y 1+ 3y1 - Sy2 = 64 sin 4t, y 2 + 5y 1- 3y2 = 12 cos 2t.
10. Yt + Y1 - 3y2 = 6e-', y 2 + 2y 1 - 4y2 = 12e'.

Find the solution of the following systems of equations .


11. (D - 2)y 1 + (D + 3)y2 = t + 4, (3D + 5)y 1 + (2D + 6)y2 ::i St+ 3.
12. (2D + 3)y1 + (D - l)y2 = e', (4D + 6)y1 + 3Dy2 = e-'.
13. (D - l)y 1 + 2Dy2 = t 2 + 1, (3D + 5)y 1 + 6Dy2 = t + 3.
14. Dy 1 + (3D - l)y 2 = e- 1, 3Dy 1 + (11D - l)y 2 = 2(e' + e-').
15. (D + 3)y 1 + (3D + 23)y2 = e- 21, (D + 2)y 1 + (4D + 14)y2 = e 2'.
16. (2D + 3)y 1 + (D + 5)y2 = t 2, (8D + 14)y1 + (11D + 28)y2 = t + 3.
17. (2D + l)y 1 + (D + l)y 2 = t, (D + 2)y 1 + (3D + 2)y2 =.2 t + 1.
18. (D - l)y 1 - (D + l)y2 = t, (D + l)y 1 + (2D + l)Y2 = e'.
Using the matrix method, find the solution of the systems of equations y' = Ay, where y = [y
1 Y2f and

19. A=[: -!1 20. -31


A=[~ -2
21.
A=[! -21
-3
22.
A=G -41
-2

23. A=[: -:1 24. A= ["2l -I]


l/2 .
Using the matrix method and the method of undetermined parameters, find the solution of the non, homogeneous
system of equations y' = Ay + h, where y = [y 1 y2]r, and

2S. A = [ ~ h- _[3' I].


+
21 + 5 26. A - [I/~
27. A = [~ 28. A = [ ~ -4]
- I ' h --
[24] 1
18 e .
5.68 Engineering Mathematics

29. A
3 2] h = [25]es'.
= [4 1' 13
. . f' d the solution of the systems.
30. In Problems 25 and 26, use the method of diagonalisauon to m

5.7 Answers and Hints


Exercise 5.1
3. Constant coeff.
1. Constant coeff. 2. Variable coeff.
6. Variable coeff.
4. Variable coeff. 5. Variable coeff.
7. Any subinterval on (- oo, 0), (0, oo). 8. Any subinterval on (- 00 • oo).
00
9. Any subinterval on (- oo, 0), (0, oo). 10. Any subinterval on [O, )-

00
11. Any subinterval on (3. oo). 12. Any subinterval on (0, )-

13. Any subinterval on (- oo, 0), (0, 1), (1, oo).


14. 4m < x < 4(m + 1). m = 0, 2, 4, ....
15. No, because the equation is not normal on any interval containingx = 0, Remark I is also not applicable.
16. 2x. No, because the equation is not normal on any interval containing x = 0.
17. No, because x = 0 at which the equation is not normal is included in the interval [- 3, 3], even though
the conditions are specified at x = 2.
21. 6x + 3 = (3/4)(2x) + (3/2)(3x + 2), linearly dependent.
22. Dependent, 9xi - x + 2 = 3(xi - x) + 2(3x2 + x + 1).
23. Independent, no linear combination can be found, alternately W = 14.
24. W = - 16 sin6 x, linearly indpendent. 25. W = I, linearly independent.
26. Dependent, W = 0, x E /. Alternately, cosh x = ex - sinh x.
27. Linearly independent, W,;. - 4/x. 28. Dependent, W = O.
29. Linearly independent, W = - 4. 30. Dependent, sinh x = cosh x _ e-x_

31. W = - 2 tan3 x. linearly independent on (0, tc/2), (<2n - 1) !:_, (2n + 1)


2
!:_)
2 • n
= 1'2, ....
32. (i) Three, (ii) Three. 33. W(y 1, Yi) = 2, y 3 = 2y 1 _ Yil2.

34. y;'= -(a 1lao )y; - (ai lao )y,, W(x) = Y1Y2 - Y2Y1. Differentiating W(x) and sub t"t .· fi *
• W'( ) 11,( ) o p· d' th • . s 1 utmg or Y · we
o b tam ao x +a,"' x = . m mg e mtegratmg factor we obtain the soluti . '·
of c depends on y 1, Yi· • on as given. The value

3S. Substitution shows that cos at, sin at are solutions. W = a ~ o y 1 1


.
interval / . Using the Abel's formula we get W = c, where c ca~ b ' tYaki are mearly independent on any
e en as a . Yes.
36. Substitution shows that e2x and xe2x are solutions of the equat· W _ "-t
·
independent on any mterva l / . U smg ton. w . - e. ~ 0 • Yt• Y2 art: 1·mearIY
· Ab e l' s formula we get w = ce"x
when c = I. htch ts same as the earlier value
112
37, Normal in (0, 00 ), W=x • (y,, Y2I forms a basis.
4
38. Normal in any /, W = 3e x. (y,, Y2) forms a basis.
39. Normal in (0, 00 ), W = 2x. (y,, Y2) forms a hasis.
Line ar Diffe renti al Equa tions 5.69
40, Normal in (- 00
' oo), W = 20. {y,, Yi, y3} forms a basis .
41. Normal in (- 00

00
), W= e3". {y,, Yi, YJ} forms a basis.
42. Normal in (- 00• 00) , W = 12.[3 . {yi, Yi, y } form
3
s a basis.
43. Normal in (0, 00) , W = - 2/x. {y1, Yi} forms a basis
.
44. W(u, v) =(ad :-- be) (Y1Y2 - YiY 1). Since y y - Yi Yi*
1 2
of th~ coeffic1e,t"t matr ix of the transformation
o, W(u, v) * o if ad- be °I' 0, (the determinant
u = e ", v = e- ". ). Take a = 1, b = 1, e = 1, d = - 1, ad - be = - 2,
45. W(y1, Y2) * 0. If for Xo E /, eithe r y 1(x0), Yi(x
vanish or y (xo), y (x ) vanish, then W(y1 , Y2) = 0.
)
1 0 2 0
46. Simplify W(y, Y1, Y2) and substitute y;' = -(ay ; +by
;), i = I , 2. We obtain
W(y, Yi, Y2) = (y" + ay' + by) (Y1Y2
- YiYD = 0.
47. At the given poin t y 1(x 1) = y'(x ) = 0. Therefore, y
1
1
= 0.
48. The diffe renti al equa tion is W(y, Yi, y ) = 0, where
2 y 1 = e3", y = e-ix, y" - y' - 6y = 0.
49. y" + 2ay' + (a 2 + w 2 )y = o. 2

50. y" - 10y' + 25y = 0.

Exercise 5.2
1. (7e" - e 4")/3.
2. (3eix - e-ix)/2.
3. (1 + 5x)e -3".
4. 1(5x 2
- (1/x 2 )).
5. (3 + ln x)x. 6. Aeix + Be-ix.
7. Aeix + Be-x. 8. Aex + Be-2x_
9. Ae6x + Be-ix . 10. Aem 1x + Bem 2 x, m1 = -2 + ..fi.
m 2 = -2 -../3 .
11. Aeix + Bex 14 . 12. A ex/2 + Be-(5x)l2.
13. (A+ Bx)e- x. 14. (A + Bx)e-nx_
15. (A + Bx)e< 2x> 13 • 16. (A + Bx)e- x 12 .
17. (A + Bx)e<2x)t5_ 18. A cos 5x + B sin 5x.
19. (A cos x + B sin x)e- 2". 20. e"(A cos x + B sin x).
21. ex 12(A cos 2x + B sin 2x). 22. e 3" (A cos 3x + B sin 3x).
23. A + Be- 9". 24. en"(A cos bx + B sin bx).
25. m = 3, _ 2, ch. equa tion is m2 - m - 6 = 0, diff.
equation is y" - y' - 6y = 0.
26. m = 114, _ 3/4, ch. equa tion is 16m2 + Sm - 3 =
0, diff. equation is 16y" + 8y' - 3y = o.
27. m = o, _ 2 , ch. equa tion is m(m + 2) = 0, diff. equa
tion-is y" + 2y' = 0.
28. m = 2, 2, ch. equa tion is (m - 2)2 = 0, diff. equa
tion is y" - 4y' + 4y = 0.
__ _ h uatio 2
29• m - 1, 1 , c . eq . n is (m + 1) = 0, diff. equation is y" + 2y' + y = 0 .
30• y ,, + 9y= 0 . 31. y" + 2ay' + (a 2 + b 2 )y = 0 .
32. y" - I0y' + 34y = 0. 33.
34. e 4" + 3e-3 x. 35.
5.70 Engin eerin g Mmh emati cs

36. a cos {i r. 37. ez,.( 2 cos ;c - 5 sin x).


- (3.1 )/ 2
38. e x'5 [cos (x/5) - sin (x/5)] . 39. ((x 12) - l) e ·
40. xe-xf3 _ 41. cos 5.x + B sin Sx, B arbitrary.
42. [ (2e2 - 1)c- 6, - "6·'] /(el - 1). 43. e-'(co s ,t + si n x) .
44. (A,· + B)e.1 13, A = " - 21 3 _ l. B = _
2 e- 21 3_
4S. (et ♦ l _ "3') /(e i. _ l ). 48. (i) b = constant. (ii) a (.x) = b(.x) .
49. (D + 4 )(0 + l )y = O. set (D + l )y = v and (D + 4)u = A _,.,. y - Ae- 4x + Be- x .
0; V = ,e ' - xn <-1.rY2
SO. (2D + 1)(2D + 3)y = 0. set (2D + 3)y = u and (2D + l)u _ A -.1n y = A C + Be .
= O, u - ,e ' -< J..r >n
51. (2D + 3)(2D + 3)y = O. set (2D + 3)y = v. (2D + 3)u A -<h>l2 y - (A.x + B) e .
= O. u = ,e · - -J..r
52. (D + 3)(0 + 3)y = 0. sel (D + 3)y =v, (D + 3)u =0 , v = 3
A, e- .. ,
. Y =(Ax + B)e
SJ. (D + 2)(D - 2)v =0, set (D - 2)y = u. (D + 2)v = 0. v A -2..r + Be h
2.x
· = A ,e- , Y = e ·
54. (3D + 1)(3D + l )y = 0. set (3D + l )y = u, (3D + l)u = - xi)
.0 , v = A,e-..r'3, Y =(Ax + B )e .
cteristic equation should be less than zero.
SS. For oscill atory solutions, the discriminant of the chara
I I - c I < 2 ✓b, I - 2 ✓b < c < l + 2 ✓b.
56. w = n. y(x) = B,. sin flt, B" arbitrary.

57. Yn(x) = A" cos ru-. A" arbitrary y(x) = L Yn (x).


n=I

58. y,, (x) = B,, sin [(2,1 + l )x/2], B,, arbitrary y (x) = -
L Yn (x).
n=I
59. y(x) = ePx(A'eqx + B'e-q x) = eP..-[A cosh qx + B sinh qx].
60. (i) For c 2 > 4mk, both the characteristic roots - p ± q where
p = c/(2m) and q = ..J c 2 - 4mk l(2m) . are
negative and q < p . Therefore, the solution y(t) = e-P'(Aeq' + Be-q'
) ➔ 0 as t ➔ 00, that is, there exists
a f<) such that for r > t0 the system is in equilibrium . y = [au
0 e-P' sinh qt]lq.

(ii) For c i < 4mk, the characteristic roots are - p ± iq, where p
= c/(2m) and q = ..J 4mk - c 2 /(2m ) are
comp lex. The solutions are oscillatory in this case. The soluti
on is y(t) = e-P'(A cos qt + B sin qt). The
oscill ations are damped and they decay as t ➔ oo. y = (e-P'u
sin qt)lq. 0
(iii) For c 2 = 4mk, the characteristic roots are repeated roots
y = Uole-P'.
- p . The solution is y(t) = (A + Br)e-P'.
61. Ae3x + Be- 2.r. 62. Aex + Be- 4x_
63. u = x + 1/x, Yi= l + x2, Ax+ B(l + x2).
64. u = - cot x , Yi= - x- 112 cos x, x- 112 (A cos x + B sin x).
6S. u =-
e-x(x i - 2x + 2), Y2 =-
(x 2 - 2t + 2), Ae·t + B(x 2 - 2t + 2).

Exercise S.3
1. A + Belx + ce-lx_ 2. Aex12 + Be 2.t + Ce - 3.t .
3. Aex + Be- x + Ce 2x 13 • 4. A e 2.t + Be- lt + Ce 1t + D - Jx
e .
S. Aex + Be ix + Ce-xii + Dexo._
6. A + Be u + ce-2t + o e - .t_
1. Ae x'" + Bexti + Ce'" + De -x. 8. A e ,.13 + Be - .t/J + Ce .t ' .., + De - .t ' " .
• f Equa tions 5.71
Linea r Differ entta
A+ (Bx+ C)ex.
9. 10. Ae- 2x +(Bx + C)e-x .
Ae-2x + (Bx+ C)e2x _
11, 12. (A +Bx+ Cr)ex fJ_
13, A+ Bex+ (Cx + D)esx_ 14. A+ (Br+ Cx + D)ex.
15, (Ax+ B)e-x + (Cx + D)ex12_
16. (Ax+ B)e 3 x + (Cx + D)e 2 x' 3 .
11. A + B cos x + C sin x. 18. Ae2x + B cos 2x + C sin 2x.
19• Ae-Jx + e-x(B cos x + C sin x). 20. Aex + eJx(B cos 2x + C sin 2x).
21. Aex + Be- x + C cos 3x + D sin 3x. 22. A ex + Be- 2x + C cos 4x + D sin 4.x.
23. A cos 5x + B sin 5x + C cos (x/2) + D sin (x/2).
24. e2x(A cos x + B sin x) + e-Jx(C cos x + D sin x).
25, (A + Bx) cos 5x + (C + Dx) sin 5x. 26. (A+ Bx) cos x + (C + Dx) sin x .
21. m = 0, 1. 3, y"' - 4y" + 3y' = 0. 28. m = - 1, ± Si, y"' + y" + 25y' + 25y = 0.
29, m = - 1, - 1, 2, y"' - 3y' - 2y = 0. 30. m = 0, 0, 1, 3, iv - 4y"' + 3/' = 0.
31, m = 2, 2, 2, - 2, iv - 4y'" + 16y' - l6y = 0.
32, m = ± 3, ± 2i, Y iv - 5y" - 36y = 0. 33. (3e3x + 2e- 2x - Sex)/3 0.
3
34. (9c - 5e x/Z + e-Jx/ 2 )/5. 35. (2 + x)ex - eJx.
36, (1 + x)e- x + (2 - x)e2x. 37. x + cos x + sin x.
38. cos 2x + 2 sin 2x - ex. 39. ex+ e-x(co s x + 2 sin x) .
40. 1 + 2x + 3x2 + e 3 x. 41. A sin rcx, A arbitra ry.
42. 1 + 2 sinh 6x + cosh 6x. 43. 2 sin 2x·+ sin 3x.
44. Dn sin nx, L Dn sin nx. 45. 2 cos 3x + cos x .

Exercise 5.4
1. A(x) = - e 2x1s. B(x) = - e- 2x1s. y = c,e-x + Cze 3·x - (ex/4).
2. A(x) = - e- 4 x14, B(x) = (4x + l)e- 4 x/16, y = (CtX + c ) e 2x
2 + e- 2 .x/16.
3. A(x) = cos3 x/3, B(x) = (sin 3x + 3 sin x)/12, Yp = (cos x)/3, y
= c 1 cos 2x + c2 sin 2x + Yr
4. A(x) = In I cos x I, B(x) = x, Yp = cos x In I cos x I + x- sin x, y
= Ct cos x + c 2 sin x + Yr
5. A(x) = - x, B(x) = In I sin x I, Yp = sin x In I sin x I - x cos x,
y = c 1 cos x + c 2 sin x + Yp-
6. A(x) = sin x - In I sec x + tan x I, B(x) = - cos x, Yp = - cos x
In I sec x + tan x I,
y = c 1 cos x + c 2 sin x + Yp-
1. A(x) = - x/2, B(x) = - e-2x14, y(x) = Ctex + c 2 e 3 x - (xex)/ 2.
8. A(x) = ! In I cos 2x I, B(x) = x/2, YP =! cos 2x In I cos 2x I + ~ x sin 2x.
y(x) = c 1 cos 2x + c 2 sin 2x + Yr
9. A(x) = (cos 4.x)/16, B(x) = (4x + sin 4x)/16, Yp = (cos 2x + 4x sin 2x)/16 .
y(x) = Ct cos 2x + c 2 sin 2x + (x sin 2x)/4.
10• A (x ) sm
=· x + x cos x
, B(x) = - cos x, Yp = - e- ix sm
. x, .Y ( )
x = ( c 1x + c 2 )e- ,,= + y ,,.
11. A(x) = - x , B(x ) = In Ix I, Yp = x [In Ix I- I] e- Jx, y(x ) = (c,x + c ) e- lx
2 +y •
11
5.72 Engine ering Mathem atics

12 A( ) - (cos 2x)/4, B(x) = (2x + sin 2x)/4, y(x) = c,e-x cos x + c2e -x sm · + (xe-x sin x)/2.
• x - x

l3. g(x) = x, A(x) = x 214. B(x) = - x 4/8, Yµ = x-118. y(x) = c 1x + (c2IX) + Yp·
14· g(x) = In I X 1. A(x) = [In I X 11 2/8, B(.x) = - x4 [ 4 In I X I - I J/64.

Yp =x 2 [8(1n Ix I )2 - 4 In Ix I+ I ]/64, y(x) =c 1.x2 + c2lx 2 +Yp·


g(x) = l/x • A(x) = [I + 5 In Ix l11(25x ), B(x) = - I /(5x ).
5
15.
6 5

Yp = l/(25x4). y(x) = c ix + cix In Ix I + Yp·


g(x) = x + ( 1/x). A(x) = - [(x /2) + In Ix I ), B(x) = x - (llx),
2
16.
Yp = (.r-'12) - x( I + In Ix I ), y(x) = c 1x + c2x + Yµ·
2

g(x) = l 6e-l r coscc 2r, A(x) = 4 In I cosec 2x + cot 2x I, B(x) = - 4/sin 2x.
2
17.
Yp = 4e-:1x cos 2t In I cosec 2x + cot 2x I - 4e- X, y(x) = e-2xcc, cos 2x + C2 sin 2.x)
2 + Yp·

18. A(x) = (In I sec 2t + tan 2r I )/8, B(x) = - x/4, C(x) = (In I cos 2x I )/8,
y(x) = c, + c2 cos 2x + c 3 sin 2x - (x cos 2x)/4 + (sin 2x In I cos 2x I )/8 + (In
I sec 2x + tan 2x I )IB.
A(x) = x /4, B(x) = - x, C(x) = (In I x I )/2,
2
19.
y(x) = (c, + c2x + c3x )e x + (x In Ix I eix)/2.
2 2 2

20. YP = ! Jofx g(t)[sin kx cos kt - cos kx sink t]dt = l


k
r
J
g(t) sin [k(x - l)]dt.
0

Exercise 5.5
5
1. Yp = - (50x2 - 30x + 69)/500, Ye= Ae-2x + Be x.
12
2. Yp = (20 - 5 lx + 9.x2 - 9x3)/27, Ye= Ae-x + Be3x •
3. Yp = (35ex + 3e3x)ll05, Ye= Aex12 + Be-x12 .
4. Yp = (e-2x - ?x - 14)/7, Ye= Ae-x + Bex 13 •
5. Yp = - e-3x + ex/15, Ye= Ae-2x + Be- 4x_
6. Yp = 3xe-x, Ye= Ae-x + Be-3x_
12
1. Yp = - xe-2x + ex/3, Ye= Ae-2x + Bex .
8. Yp = 2xe3x - xe-2x, Ye= Ae-2x + Be3x.
9. Yp = 2xex13, Ye= Ae-2x + Bex/3_
10. Yp = (2 sin x - cos x)/5, Ye = Ae-x + Be-ix.
2 3
11. Yp = (sin 3x - 5 cos 3x)/2, Ye= Ae x + Be- x_
12. Yp = 2(sin 2x - cos 2x), Ye= Aex + Be-sx_
13. Yp = x(- 3 cos 5x + 5 sin 5x), Yr= A cos 5x + B sin 5x.
14. Yp = - 2x cos 4x, Ye = A cos 4x + B sin 4x.
15. Yp = 4.x2e2x + e3x, Ye = (Ax+ B)e2x.
12
16. Yp = 3x 2/xl2>;4, Ye = (Ax + B)ex •
3
17. Yp = l3x2e-Jx + e2x/5, Ye= (Ax+ B)e- x.
18. Yp = ex(sin x - 2 cos x)/5, Yr= A cos x + B sin x.
19. Yp = - (xe-x cos 3.x)/6, Yr= e-x(A cos 3x + B sin 3x).
2
20. Yp = 8xe2x sin x, y,. = e r(A cos x + B sin x) .
Linear Differential Equations 5.73
3
21. Yp = - 3xe x cos 2xl4, Yr = e 3x(A cos 2x + B sin 2x).
22. r (x) = 3e-2x ( l + cos 2x), Yp = e-2x <c1x2 + Cz cos 2x + C3 sin 2x) = [3e-2x (2x2 - cos 2x) ]14, Ye= (Ax+ B)e-2.x.
23. r(x) = 3e-x( 3 sin x- sin 3x), Yp = e-x[-45(cos x + sin x) .+ (cos 3x + 3 sin 3x)]IIO,yc = Ae-x + Be-2.x
3 3
24. r (x) = 2(e x + e- x), Yp = (e- 3x + 12xe3x)l 12, Yr = A ex + Be 3 x.
25. Yp = - 3xe-x, Yr= Aex + Be-x + Ce-4x_
2
26. Yp = xex - 2x e-2x, Yr = (Ax+ B)e-2.x + Cex.
3
21. Yp = 6x3e x, Yr= (Ax 2 + Bx+ C)e3x.
28. Yp = 2(cos 2x - 2 sin 2x)/5, Ye= Aex + B cos x + C sin x.
29. Yp = - [2(x2 + x) + x(cos 2x + sin 2x)]l2, Ye= Ae2x + B cos 2x + C sin 2x.
30. Yp = - x sin 4xl2, Ye= Ae4x + Be-4x + C cos 4x + D sin 4x.
4
31. Yp = - (x + 25), Ye= Aex + Be-x + C cos x + D sin x.
2
32. Yp = x - 2x, Ye= A+ (Bx 2 + Cx + D)e-x.
2
33. Yp = 3xe x, Ye = Ae 2x + Be- 2x + C cos X + D sin x.
34. Yp = - 5x e-2x, Ye= A+ (Bx2 + Cx + D)e- 2x.
3

35. Yp = - (x3 + 6x 2)112, Ye= Ax+ B + Ce4x + De- 4 x.

Exercise 5.6

1. y = Ax2 + Blx2. 2. y = (Alx) + (Blx2).


3. y = Ax + Blx. 4. y = (A + B In x)x- 113•
5. y = (A + 1J In x)x-312 • 6. y = A cos (In xl.fi) + B sin (ln xl../2).
1. y = (A + B In x)lx. 8. y = x[A cos (2 In x) + B sin (2 In x)].
1
9. y = x- [A cos (3 In x) + B sin (3 ln x)].
10. y =x 113 [A cos (In x) + B sin (In x)]. 11. y =A + Bx + C In x.
2
12. y = [A + B In x + C ln x]x. 13. y =Ax+ x- 1[8 cos (In x) + C sin (In x)].
14. y = (Alx) + (Blx2) + (Clx3). 15. y = (Alx) + (B + C In x)x2•
16. y = (Alx2) + x[B cos (4 In x) + C sin (4 In x)].
2
11. y =A+ Bx+ Cx + D In x. 18. Y = Ax2 + (Blx2) + C cos (ln x) + D sin (ln x).
19. y = A,/x + (Bl,/x) + C cos (2 In x) + D sin (2 In x).
20. y = (A + B In x)x + (C + D In x)lx.
21. y = Ax2 + (Blx) - x - 3. 22. y = Ax + Bx3 + In x + 2.
2
23. y=Ax+(Blx2)+2xlnx+7. 24. y = Ax + (Blx') + 3x2 In x.
25. y = A + (Blx) + [sin (In x) - cos (ln x)]l2.
26. y = Ax + (Bir) + 2x(3 ln 2 x - In x)l3.
27. y = (A + B ln x)x 112 + 4 cos (ln x) - 3 sin (ln x).
28. y = (A + B ln x)x2 + x3. 29. y = (A + B In x)x- 3/:? + 2 sin (In x) - cos (ln x).
30. y =Ax+ (Blx 2 ) - x[3 cos (In x) + sin (In x)]/10.
31. y = (Alx) + Bx4 - x 2 - In x + 314. 32. Y =Ax+ (Blx) + (C/.\.5) + 2\'.!_
5 -74 Engineering Mathematics

2
33. y = Ax + (Blx 2) + (C/x:3) - (3 In x)lx2.
34. Y = (A + B In x + C ln 2 x)x 2 + 3x3 - 8x.
1
35. Y =(A+ B In x)x 112 + (Clx) + sin (In x) + 7 cos (1n x) .

36. Set3x+l= z.y = (A+Bln(3x+l)l(3x+1 ) 113 + ~(x-1).

37. Set x + 2 = z. y = A(x + 2) + (x + 2) 112[8 cost+ C sin t] + 8(x + 2)2 - 9 6 (x + 2) In (x + 2) - 96 •


where t = ../3 In (x + 2)/2.
38. y =Ax+ (Blx) + Cx2 + (Dlx2) + l/(4x3).
39. y = Ax3'2 + BX-312 + (C + D In x)x + 2x2 - 1/9.
40. Y = A cos (In x) + B sin (In x) + C cos (2 In x) + D sin•(2 In x) + 1/(20x2).

41. y = ! (rx + ~) + ~.
42. y = 4(ln x - 1) ,,fx + In x + 4.
43. y = [1x - 10x2 + 5x3 + x In x]/2.
44. y = x[4 sin (In x) - 2 cos (ln x)] + 3.

45. y = ! [2 cos (3 In x) + 3 sin (3 In x) + x; .


Exercise 5. 7
2. Aex + Be-x + e 3x.
3. Ae-x + Be4x + e5x - (ex)/6. 4. e-x/2 [A cos ( .,ffx/2) + B sin ( ..ffx/2)] + Ii ex/2.

5. e- 3 xii [A cos ( ../3xl2) + B sin ( .JJx/2)] + ex.


6. (A + Bx)ex + 4e 2 x + (5e4x)/9. 7. (A + Bx)ex 13 + (e-~/4.
8. (A + Bx)e3x + 1x 2e 3 x. 9. Ae2x + Be-Jx + (xe 2 x)/5.
10. Ae2x + Be-x 12 - e-x/2 (4x + 5x2)150. 11. Aex + Be-x + [3ex(x 2 - x)]/2.

12. Ae-2x + Be-x/4 1 (7x 2 + 8x)e- 2 x.


-
98 13. (A + Bx)e-x/3 + (x2 e-x 13)/18.

14. Aex/2 + Be.:. 4x - e- 4x (9x2 + 4x)/162.


15. Ae-x + Be 2x + Ce- 3 x - (ex)/2. 16. Aex +Be-ix+ ce-xn + (ei...)12.
17. Aex + Be-x + Ce 2x + (e 3x)/8. 18. (A+ Bx+ Cx 2 )e2x + 3x3e2x.
19. (A + Bx)ex + Ce-xi~ + (8x 2ex)/3. 20. Ae2x + Be-lt + Ce-Jx - 3e-2x (2x2 _ 3x)/4 .
21. A cos 4x + B sin 4x + (cos 2x)/12. 22. Aex + BeJx12 + (sin x + 5 cos x)/26.
2 3, Ae2x + Bex/3 + (3 cos x - 4 sin x)/25. 24. AeJx + Bex 12 + (14 cos -,t' _ 5 sin
2 t)/2 2 1.
25. e- xl2[A cos (../3xl2) + B sin (./3xl2)] + 16 sin x.
26. e3.r/4[A cos (x/4) + B sin (x/4)] + 16(4 cos x - sin x)/51.

21. A cos 3x + B sin 3x - (x cos 3x)/6. 28. A cos ( ../3x) + B sin ( ../3x) + (x sin ..fSx)/(2../3).
Lir1<'llr D{ff't·n,11rial Eq11mim1s 5.75

29. , - l(A ros 2x + B sin 2x) + {.u-r sin l r)/4.


30. r:\ .-\ cos x + B sin x ) - 12. x C'OS x e .tr_

31. r\r(A ros 2x + B sin 1-x) - 7x cos 2r ,.t...


32- r'(A ros lr + B sin l ,· + x (S sin lr- 12 cos lr)/) J.
3.\ Aru + B rns x + C sin x - 1\"(cos x + 3 sin x)/10.
1
3,$. Ar + 8 cos l r + C sin lr - x(3 cos lx + sin lr)n.
35. Ar1r + ex(B cos 2x + C sin 2t) - 6.u c (2 sin lr - cos lt)/5.
36. Ae!T + e xr.(8 cos x + C sin x) - 4xe rf2(2 cos x + 3 sin x)/ 13.
37 . .-\ cos x + B sin x + ~ cos l t + D* sin l r - Sx(cos x + 2 sin l \·)/3.
JS. A cos 5x + B sin 5x + (225x' + lO(lx.! - 54x .. 8)/625.
39. (A+ Bx)e-lr + (12,1 - 16.\· + 5)127 .
.w. 1
At-cl + Be T - (18.x1 + 3Qr - 8)n7 .
1
.n. Ae:u- + Be .. + [(52x + 25)(cos lx - 5 sin lt) - 21(5 cos l,· + sin 2r)Jn704.
42. Atx + Be-z.-c - [(25.t2 + Sx - 9)(3 sin x + cos x) + (35x + 12)(3 cos .t - sin x)]/250.
.O. Aelr + Be- 2.. - e-2t(5.t2 + l.t)/50.
44. Ae-3..t + Be-t:c + ex(S sin lt - 9 cos lt)n90.
45. Ae-:c + Be-3.t + e2t(7 cos x + 4 sin x)/130.

46. e-l.tr.?[A cos p + B sin p] + 4ex(25 cos p + 10~ sin p)/1325. p = ~x/2.
47. Write xex sin x = Im [xil+i).r], Ae-x + Be-lt + ex[S(l - x) cos x + (5x - 2) sin x]/50.
2
48. Write xe 2t cos x = Re [xe<2+i).r], A cos 3x + B sin 3x + e t[(3Ut- 11) cos x + (lOx- 2) sin x]/400.
49. Ae-.xn + Be- 3:t1i - e-:c1i [(x - 2) cos x - (x + 1) sin x]/8.
2
50. A cos x + B sin x + ~ cos ..fix + D* sin ..fix - 4[9x cos x - (2x3 - 5 lx) sin x]/3.

51. y =Aex12 + Be1\ B = 115.

52. J,-.,r(x)dx = J,-•x (D- m)ydx =,-"'y, or Y. =e"' J,-•x r(x)dx .


53. Use the result

d
-d
X
f b db
f(x , t)dt =f(x, b) -d
X
da
-f(x, a)-d. +
.X
f -aaJ.
a
b
X
dt
a

2d =
f x
r(t) cos n(x- t)dt, --?
d2
= r(x) -
fb r(t) sin 11(x - t)dt = r(x) - ,, 2 y.
1l
dx dx·
• a
a

54. D'"(x 11) =xD'"u + mD'"- 11 = xD'"11 + [:DD'" ]11 m = 1, 2, ...


1

F(D)(x u) = x[a 0 D" + a 1D


11
-
1
+ . . . + a,.]11 + /v [aoD" + a1D"- 1+ .. . + a 11 l11

= xF(D)u + F' (D)u.


55. F(D)(xu) =x F(D)u + F'(D)u . Let F(D)u =11 .
F(D)lx{F (D)r
1
11] =xF(D)[F (D)r 111 + F'(D)lF( D)r1u =x11 + F'(D)lF( D)r 'u
5 .76 Engin eering Math n,wti cs

xu = F(D) [x{F( D)r 1u] - F'(D) [F(D )r 1u


[F(D )r 1(xu) = x[F(D )r 1u - F'(D) [F(D )r 2u.
56. When F(m) = O. F'(m) ~ O. F(D) (D - m)G( D) and =
F'(m)

[F(D )r' emx = ~~=) =G(m) .


=x ( F,:m ) )e'"x .
When F(m) = 0, F'(m) = O, F"(m ) ~ 0, F(D) = (D - m) 2G(D) d F"(m ) = 2G(m).
an

[F(D) ]-1 e'"x =


xi e'"x
2! G(m) =x
2( F"(m
1 )e'"x.
)
Probl em 8: F(3)= O. F'(3) = 0, F"(3) = 2, Yp = 7x 2e 3x·
Probl em 9: F(2) = 0, F'(2) = 5, Yp = (xe4l 5.
Probl em 13: F(- 1/3) = 0, F'(- 1/3) = 0, F"(- 1/3) = 13
18, Yp = (x2e-x )/lS.
Probl em 19: F(l) = 0, F'(l) = 0, F"(l) = 6, Yp = (8x 2
ex)/3.
57. Note that [F(D )r 1 can be written as
[F(D )r 1 = A1(D - m1)- 1 + Ai(D - m2f 1 + . .. + AnCD
- m,,r 1
· 1 1 A1 A2
( eqmv alent to writing in partial fractions of F(m) as F(m) = m _ m, + m _ m + · · · + m All
- m
)
2 11

Now apply the soluti on of Probl em 52.

Probl em 40: YP = ! e3x f e-3x (2x 2 + 6x)d x - ! e-x f ex (2x 2 + 6x)d x

= - i? (l8x 2 + 30x - 8).


58. For force d damp ed oscill ations , c 2 < 4mk, Ye= e-c,icim>.
[A cos dt + B sin dt]

d = ✓4mk - c 2 /(2m) .

For force d undam ped oscill ations , Ye= A cos (,Jk!m t) + B


sin (,Jk!m t).
c -:t:- 0, Yp = F 0 [(k - mo>2) cos wt+ cw sin wt]l[ (k- mw2
)2 + c 2 2 w ]
c = 0, Yp = F0 cos wtl(k - mw 2 ) .

Exer cise 5.8


1. y "1 - 4 Y '1 + 3 Y1 -- 0 , Y1 -- A e' + B e 31, Y2 - B e 31 - A e 1.

2. yt' - 2y 1- 8y 1 = 0, Y1 = Ae- 21 + Be41 , y 2 = 3Be41 - 3Ae-21•


3. y 1 =A cos 3t + B sin 3t, Y2 =3.( B cos 3t -A sin 3t).
4. y 1 = Ae- 1 + Be- 41, y 2 = - (3Ae- 1 + 6Be- 4').
S. .y 1 = Ae 1 + Be- 1 + 2 cos t, Y2 = Be- 1-Ae 1+ 6 sin t.
6. y 1 = Ae 51 + Be- 3 ' + 4e- 1l3 + 8(15t - 2)/75.
3y = 4e- 1 - 6Ae 5' + 2Be- 31 - 8(15t + 13)/75 .
2
7. y 1 = Ae 21 + Be- 2' + 4e 1 + e- 1, Y2 = - (5Ae 2 ' + Be- 21 + 10e'
+ 2e-1).
Linear D~ffere11tia/ Equations 5.77

s. y 1 = A cos 31 + 8 sin 3t + (l/2)e' - 8 sin r + 2 cos 1.


5y2 = (48 - 3A) sin 3t + (38 + 4A) cos 3t + (5/2)e' - 50 sin 1.

9, y 1 = A cos 4t + 8 sin 4t + 5 cos 21 + 321 sin 41 + 241 cos 41.


5y2 = (38 - 4A) sin 4t + (48 + 3A) cos 4i - 10 sin 21 + 15 cos 21 - 32 sin 41 + (24 + 2001) cos 41.
2
10. YI = Ae I + Be ' - se-l - 361e', 3y2 = 2Ae I + 3Be 2' - 6e- 1 - 36(1 + 2l)e'.
9
11, y 1 = Ae- ' + (9t - 16)/27, Y2 = - (11/6) Ae-9' + (1/81) (52 + 451) + Be-3' .
12, y 1 = (l/5)e' + 2e-' - 3Ae- 2 ' + Be- 3' 12 • Y2 = Ae-21 + e- 1 - .(2/3)e'.
2
13, Y1 = (t - 31 )/8, Yi= A + (10z3 + 2Ir2 + 421)/96.
14. y 1 = [(4t - I - 8A)e-' - 2e']/2, y 2 = [(2A - 1)e- 1 + e']/2.
2 4
15. y I = (29/6)e ' - 5Ae ' - lOBe- 1 + e- 21 , y 2 = Ae4' + Be-1 - (5/6)e 2'.
12
16. y 1 = - (9/4)Ae-' + 3Be- 2' + (1413 - 7381 + 16812)/84.
12
Y2 = Ae-' + Be- 2' - (207 - 1141 + 2812)/28.
17. Y1 = - (1 +A)+ (l/3)Be- 4 ' 15 , y2 =A+ Be-4115 + t.
18. Y1 = [e' - 2A - Be-'13 - 6 - 2t + r]/2, y 2 = A + Be-113 + 2t - (r/2).
19. y = Ce 21x0 > + De31x<2>, x0 >= [- 2, lf, x< 2> = [- 1, lf.
20. y = ce- x<l) + De x< 2>, x<l) = [1, 2]7, x< 2> = [3, If.
1 41

21. y = ce3;, x<n + De-3;, x<2>, x<l) = [I + i, 3]7, x< 2> = [I - i , 3]7, or as
y = Cz0 >+ Dz(2), zO> = [cos 3t - sin 3t, 3 cos 3tf, z< 2>= [cos 31 + sin 3t, 3 sin 3t].
22. y =A 1e2it x0 > + B 1e_2;' x< 2>, x<O = [2, 1 - if, x< 2> = [2, 1 + if, or as
y = czO> + Dz< 2>, zO> = [2 cos 2t, cos 2t + sin 2tf, z< 2> = (2 sin 2t, sin 21 - cos 2tf.
23. y = Ce' zO> + De' z<2>, zO> = [cos t, sin tf, z< 2>= [sin t, - cos rf.
24. y = e'12 cezo> + Dz< 2>), z0 >, z< 2>as in problem 23.
25. y = c 1e21 xO> + c 2 e- 31 x< 2>-d1- e, x< 1> = (1, 1]7, x< 2> = [I, -4]7, d = [4/3, 5/3]7, e = [17/9, 4/9]r.
26. y = c 1~ x<O + c 2 e-112 x< 2> + de 2', x< 1> = [3, lf, x< 2> = [I, If, d = [- 12/15, 28/15].
27. y = c 1e21 x< 1>+ c 2 e-31 x< 2 >+ (dt + e)e21, x0 >= [6, If, x< 2> = [I, If. d = [-6/5, - I/Sf, e = [- 16/5, Of.
28. y = c 1e'x< 1>+ c 2 e 31 x< 2 >+ (dt + e)e', x< 0 = [I, If, x< 2> = [2, If, d = [12, I2f, e = [-3, Of.
29. y=c 1e-1 x< 1>+c2e5'x< 2>+(d1+e)e5',x<l)=[l,-2f,x<2>=[l, If,d=[21, 2If.e=[2. Of .
9
30. Problem25: x=[: 1
-~Jg=x- h=~[:~; JA 1=2,-l 2 =-3,
u; = A.1u1 + g1, u 1 = c 1e 2' - (71 + 8)/5, u2 = c 2 e-3' + (31 - 13)/45,

c 1e 2' + c 2 e- 3' - (12t + 17)/9]


Y =XU=
[ c 1e 2 ' - 4c 2 e- 3, - (151 + 4)/9

Problem 26: x = [! :Jg= x- h =[-:],••, A1 = 1/2, A,= -1/2.


1

, 1 t/2 - (4/3) e 2, ,u =c e _,,2 + ( 16/5)e,


~,
u1=/1, 1u1 +g1,u 1 =c 1e 2 2
12 112 2
3c 1e' + c 2e - - (4/5 )e ' ]
Y=XU= ~ .
[ c 1e' 12 + c 2 e - ' 12 + (28/15)e·'

You might also like